emacs-orgmode@gnu.org archives
 help / color / mirror / code / Atom feed
* Bug: Orgmode export takes "AC:" as a link keyword [9.5 (nil @ /Users/junwei/.emacs.d/.local/straight/build-27.1/org-mode/)]
@ 2020-12-11 17:21 Junwei Wang
  2020-12-13  6:05 ` Kyle Meyer
  0 siblings, 1 reply; 4+ messages in thread
From: Junwei Wang @ 2020-12-11 17:21 UTC (permalink / raw)
  To: emacs-orgmode

[-- Attachment #1: Type: text/plain, Size: 149109 bytes --]



Remember to cover the basics, that is, what you expected to happen and
what in fact did happen. You don't know how to make a good report? See

https://orgmode.org/manual/Feedback.html#Feedback

Your bug report will be posted to the Org mailing list.
------------------------------------------------------------------------

When exporting orgmode file into HTML file, the exporter "mistakely"
consider "AC:" as some keyword for linking and the string following
"AC:" would be something link to.
I tried "ABC:" or "AB:", the exporter does not have this issue.

for example, code below
"""
AC:123
AB:123
ABC:123
"""
will generates

example

Emacs : GNU Emacs 27.1 (build 1, x86_64-apple-darwin19.6.0, NS 
appkit-1894.60 Version 10.15.7 (Build 19H2))
of 2020-11-11
Package: Org mode version 9.5 (nil @ 
/Users/junwei/.emacs.d/.local/straight/build-27.1/org-mode/)

current state:
==============
(setq
org-footnote-auto-label 'plain
org-roam-db-location "/Users/junwei/.emacs.d/.local/etc/org-roam.db"
org-src-mode-hook '(org-src-babel-configure-edit-buffer 
org-src-mode-configure-edit-buffer
doom-modeline-set-org-src-modeline)
org-fontify-whole-heading-line t
org-capture-prepare-finalize-hook '(org-roam-capture--install-finalize)
org-roam-dailies-calendar-hook '(org-roam-dailies-calendar-mark-entries)
org-link-shell-confirm-function 'yes-or-no-p
org-mode-local-vars-hook '(eldoc-mode)
org-babel-after-execute-hook '(org-redisplay-inline-images)
org-insert-heading-respect-content t
org-after-refile-insert-hook '(save-buffer)
outline-view-change-hook '(minimap-sync-overlays)
org-metadown-hook '(org-babel-pop-to-session-maybe)
org-roam-title-to-slug-function 'org-roam--title-to-slug
org-clock-out-hook '(org-clock-remove-empty-clock-drawer)
org-refile-targets '((nil :maxlevel . 3) (org-agenda-files :maxlevel . 3))
org-html-format-inlinetask-function 
'org-html-format-inlinetask-default-function
org-roam-verbose nil
org-enforce-todo-dependencies t
org-roam-mode-hook '(org-roam-bibtex-mode)
org-ref-create-notes-hook '((lambda nil (org-narrow-to-subtree)
(insert
(format "cite:%s\n" (org-entry-get (point) "CUSTOM_ID")))
)
)
org-odt-format-headline-function 'org-odt-format-headline-default-function
org-ref-show-broken-links nil
org-special-ctrl-a/e t
org-imenu-depth 8
org-roam-bibtex-mode t
org-agenda-files '("~/github/org/")
org-ascii-format-inlinetask-function 'org-ascii-format-inlinetask-default
org-ref-pdf-directory "~/github/biblio/papers"
org-reveal-start-hook '(org-decrypt-entry)
org-modules '(ol-bibtex)
org-blocker-hook '(org-block-todo-from-children-or-siblings-or-parent)
org-roam-buffer-prepare-hook '(hide-mode-line-mode 
org-roam-buffer--insert-title
org-roam-buffer--insert-backlinks
org-roam-buffer--insert-ref-links)
org-mode-hook '(er/add-org-mode-expansions 
+lookup--init-org-mode-handlers-h
(closure (t) (&rest _)
(add-hook 'before-save-hook 'org-encrypt-entries nil t))
#[0 "\301\211\207" [imenu-create-index-function org-imenu-get-tree] 2]
doom--setq-gcmh-high-cons-threshold-for-org-mode-h
#[0 "\300\301\302\303\304$\207"
[add-hook change-major-mode-hook org-show-all append local] 5]
#[0 "\300\301\302\303\304$\207"
[add-hook change-major-mode-hook org-babel-show-result-all append local]
5]
org-babel-result-hide-spec org-babel-hide-all-hashes
(closure (t) nil
(progn (set (make-local-variable 'time-stamp-active) t)
(set (make-local-variable 'time-stamp-start) "#\\+LAST_MODIFIED:[ ]*")
(set (make-local-variable 'time-stamp-end) "$")
(set (make-local-variable 'time-stamp-format)
"[%Y-%02m-%02d %3a %02H:%02M]")
)
(add-hook 'before-save-hook 'time-stamp nil 'local))
doom-disable-show-paren-mode-h doom-disable-show-trailing-whitespace-h
+org-enable-auto-reformat-tables-h +org-enable-auto-update-cookies-h
+org-make-last-point-visible-h evil-org-mode toc-org-enable writegood-mode
embrace-org-mode-hook org-eldoc-load)
org-roam-mode t
org-clock-persist 'history
org-ref-insert-cite-function 'org-ref-helm-insert-cite-link
org-export-with-smart-quotes t
org-odt-format-drawer-function #[514 "\207" [] 3 "\n\n(fn NAME CONTENTS)"]
org-outline-path-complete-in-steps nil
org-archive-hook '(org-attach-archive-delete-maybe)
org-agenda-finalize-hook '(+org-exclude-agenda-buffers-from-workspace-h
+org-defer-mode-in-agenda-buffers-h)
org-startup-indented t
org-roam-title-change-hook '(org-roam--update-file-name-on-title-change
org-roam--update-links-on-title-change)
org-roam-capture-function 'org-capture
org-ref-clean-bibtex-key-function '(lambda (key) 
(replace-regexp-in-string ":" "" key))
org-agenda-before-write-hook '(org-agenda-add-entry-text)
org-metaup-hook '(org-babel-load-in-session-maybe)
org-noter-notes-search-path '(org-directory "~/github/org/roam/biblio")
org-bibtex-headline-format-function #[257 "\300\236A\207" [:title] 3 
"\n\n(fn ENTRY)"]
org-latex-format-drawer-function #[514 "\207" [] 3 "\n\n(fn _ CONTENTS)"]
org-agenda-deadline-faces '((1.001 . error) (1.0 . org-warning)
(0.5 . org-upcoming-deadline)
(0.0 . org-upcoming-distant-deadline))
org-babel-pre-tangle-hook '(save-buffer)
org-tab-first-hook '(+org-yas-expand-maybe-h +org-indent-maybe-h
org-babel-hide-result-toggle-maybe org-babel-header-arg-expand
+org-cycle-only-current-subtree-h +org-clear-babel-results-h)
org-hide-leading-stars t
org-ref-clean-bibtex-entry-hook '(org-ref-bibtex-format-url-if-doi 
orcb-key-comma
org-ref-replace-nonascii orcb-& orcb-%
org-ref-title-case-article orcb-clean-year orcb-key
orcb-clean-doi orcb-clean-pages orcb-check-journal
org-ref-sort-bibtex-entry orcb-fix-spacing)
org-load-hook '(+org-init-org-directory-h +org-init-appearance-h 
+org-init-agenda-h
+org-init-attachments-h +org-init-babel-h +org-init-babel-lazy-loader-h
+org-init-capture-defaults-h +org-init-capture-frame-h
+org-init-custom-links-h +org-init-export-h +org-init-habit-h
+org-init-hacks-h +org-init-keybinds-h +org-init-popup-rules-h
+org-init-protocol-h +org-init-protocol-lazy-loader-h
+org-init-smartparens-h org-roam-mode)
org-link-abbrev-alist '(("doom-repo" . 
"https://github.com/hlissner/doom-emacs/%s")
("wolfram" . "https://wolframalpha.com/input/?i=%s")
("wikipedia" . "https://en.wikipedia.org/wiki/%s")
("duckduckgo" . "https://duckduckgo.com/?q=%s")
("gmap" . "https://maps.google.com/maps?q=%s")
("gimages" . "https://google.com/images?q=%s")
("google" . "https://google.com/search?q=")
("youtube" . "https://youtube.com/watch?v=%s")
("github" . "https://github.com/%s"))
org-export-backends '(md ascii html icalendar latex odt)
org-roam-completion-everywhere t
org-ref-insert-label-function 'org-ref-helm-insert-label-link
org-roam-capture-templates '(("d" "default" plain 
#'org-roam-capture--get-point "%?"
:file-name "%<%Y%m%d>-${slug}" :head
"#+TITLE: ${title}\n#+CREATED: %U\n#+LAST_MODIFIED: 
%U\n#+ROAM_ALIAS:\n#+ROAM_TAGS:\n\n" :unnarrowed t)
("D" "default copy" plain #'org-roam-capture--get-point "%?"
:file-name "%<%Y%m%d%H%M%S>-${slug}" :head
"#+TITLE: ${title}\n#+CREATED: %U\n#+LAST_MODIFIED: %U\n\n"
:unnarrowed t)
)
org-ref-open-pdf-function 'org-ref-open-pdf-at-point
org-clock-in-resume t
org-ascii-format-drawer-function #[771 "\207" [] 4 "\n\n(fn NAME 
CONTENTS WIDTH)"]
org-roam-graph-viewer "/Applications/Firefox.app/Contents/MacOS/firefox"
org-attach-store-link-p t
org-list-demote-modify-bullet '(("+" . "-") ("-" . "+") ("*" . "+") 
("1." . "a."))
org-archive-subtree-save-file-p t
org-agenda-span 10
org-ref-open-notes-function '(lambda nil (org-show-entry) 
(outline-show-branches)
(outline-show-children) (org-cycle '(64))
(recenter-top-bottom 0))
org-agenda-loop-over-headlines-in-active-region nil
org-src-lang-modes '(("redis" . redis) ("php" . php) ("arduino" . arduino)
("md" . markdown) ("C" . c) ("C++" . c++) ("asymptote" . asy)
("bash" . sh) ("beamer" . latex) ("calc" . fundamental) ("cpp" . c++)
("ditaa" . artist) ("dot" . fundamental) ("elisp" . emacs-lisp)
("ocaml" . tuareg) ("screen" . shell-script) ("shell" . sh)
("sqlite" . sql))
org-src-preserve-indentation t
org-fontify-quote-and-verse-blocks t
org-roam-completion-system 'ivy
org-roam-backlinks-mode-hook '(turn-on-visual-line-mode)
org-catch-invisible-edits 'smart
org-occur-hook '(org-first-headline-recenter)
org-agenda-mode-hook '(+org-habit-resize-graph-h evil-org-agenda-mode)
org-agenda-window-setup 'current-window
org-protocol-protocol-alist '(("org-roam-file" :protocol "roam-file" 
:function
org-roam-protocol-open-file)
("org-roam-ref" :protocol "roam-ref" :function
org-roam-protocol-open-ref)
)
org-ref-bibtex-assoc-pdf-with-entry-move-function 'rename-file
org-ref-insert-link-function 'org-ref-helm-insert-cite-link
org-ref-insert-ref-function 'org-ref-helm-insert-ref-link
org-indirect-buffer-display 'current-window
org-cycle-hook '(org-cycle-hide-archived-subtrees org-cycle-hide-drawers
org-cycle-show-empty-lines org-optimize-window-after-visibility-change)
org-todo-keywords '((sequence "TODO(t)" "PROJ(p)" "STRT(s)" "WAIT(w)" 
"HOLD(h)" "|"
"DONE(d)" "KILL(k)")
(sequence "[ ](T)" "[-](S)" "[?](W)" "|" "[X](D)"))
org-speed-command-hook '(org-speed-command-activate 
org-babel-speed-command-activate)
org-tags-column 0
org-ref-default-bibliography '("~/.cb/cryptobib/crypto.bib" 
"~/github/biblio/thesis.bib"
"~/github/biblio/misc.bib" "~/github/biblio/tifs.bib"
"~/github/biblio/crxwork/wheely.bib")
org-capture-mode-hook '(+org-show-target-in-capture-header-h 
evil-insert-state)
org-clock-persist-file 
"/Users/junwei/.emacs.d/.local/etc/org-clock-save.el"
org-attach-id-dir "/Users/junwei/github/org/.attach/"
org-roam-server-mode t
org-M-RET-may-split-line nil
org-odt-format-inlinetask-function 
'org-odt-format-inlinetask-default-function
org-latex-prefer-user-labels t
org-ref-pdf-to-bibtex-function 'copy-file
org-publish-timestamp-directory 
"/Users/junwei/.emacs.d/.local/cache/org-timestamps/"
org-display-remote-inline-images 'download
org-refile-use-outline-path 'file
org-priority-faces '((65 . error) (66 . warning) (67 . success))
org-export-before-parsing-hook '(org-attach-expand-links)
org-font-lock-set-keywords-hook '(doom-themes-enable-org-fontification)
org-id-locations-file "/Users/junwei/github/org/.orgids"
org-confirm-shell-link-function 'yes-or-no-p
org-link-parameters '(("printindex" :follow org-ref-index :export
#[(path desc format) "\b\301=\205 \300\302!\207"
[format latex "\\printindex"] 2]
)
("index" :follow #[(path) "\301\b!\207" [path occur] 2] :export
#[(path desc format) "\b\302=\205\n\300\303 \"\207"
[format path latex "\\index{%s}"] 3]
)
("bibentry" :follow
(lambda (_) (funcall org-ref-cite-onclick-function nil)) :export
org-ref-format-bibentry :complete org-bibentry-complete-link
:help-echo
(lambda (window object position)
(when org-ref-show-citation-on-enter
(save-excursion (goto-char position)
(let
((s (org-ref-format-entry (org-ref-get-bibtex-key-under-cursor)))
)
(with-temp-buffer (insert s) (fill-paragraph) (buffer-string)))
)
)
)
:face org-ref-cite-link-face-fn :display full :keymap
(keymap
(tab lambda nil (interactive)
(funcall org-ref-insert-cite-function))
(S-up . org-ref-sort-citation-link)
(S-right lambda nil (interactive) (org-ref-swap-citation-link 1))
(S-left lambda nil (interactive) (org-ref-swap-citation-link -1))
(C-right . org-ref-next-key) (C-left . org-ref-previous-key)
(16777337 lambda nil
"Paste key at point. Assumes the first thing in the killring is a key."
(interactive) (org-ref-insert-key-at-point (car kill-ring)))
(16777303 lambda nil "Copy all the keys at point." (interactive)
(kill-new (org-element-property :path (org-element-context))))
(16777335 lambda nil (interactive)
(kill-new (car (org-ref-get-bibtex-key-and-file))))
(16777318 lambda nil (interactive)
(save-excursion (org-ref-open-citation-at-point)
(kill-new (org-ref-format-bibtex-entry-at-point)))
)
(16777319 . org-ref-google-scholar-at-point)
(16777317 lambda nil "Email entry at point" (interactive)
(org-ref-open-citation-at-point) (org-ref-email-bibtex-entry))
(16777315 . org-ref-wos-citing-at-point)
(16777330 . org-ref-wos-related-at-point)
(16777326 . org-ref-open-notes-at-point)
(16777328 . org-ref-open-pdf-at-point)
(16777333 . org-ref-open-url-at-point)
(16777314 . org-ref-open-citation-at-point)
(16777327 . org-ref-cite-hydra/body) (follow-link . mouse-face)
(mouse-3 . org-find-file-at-mouse) (mouse-2 . org-open-at-mouse))
)
("Autocites" :follow
(lambda (_) (funcall org-ref-cite-onclick-function nil)) :export
org-ref-format-Autocites :complete org-Autocites-complete-link
:help-echo
(lambda (window object position)
(when org-ref-show-citation-on-enter
(save-excursion (goto-char position)
(let
((s (org-ref-format-entry (org-ref-get-bibtex-key-under-cursor)))
)
(with-temp-buffer (insert s) (fill-paragraph) (buffer-string)))
)
)
)
:face org-ref-cite-link-face-fn :display full :keymap
(keymap
(tab lambda nil (interactive)
(funcall org-ref-insert-cite-function))
(S-up . org-ref-sort-citation-link)
(S-right lambda nil (interactive) (org-ref-swap-citation-link 1))
(S-left lambda nil (interactive) (org-ref-swap-citation-link -1))
(C-right . org-ref-next-key) (C-left . org-ref-previous-key)
(16777337 lambda nil
"Paste key at point. Assumes the first thing in the killring is a key."
(interactive) (org-ref-insert-key-at-point (car kill-ring)))
(16777303 lambda nil "Copy all the keys at point." (interactive)
(kill-new (org-element-property :path (org-element-context))))
(16777335 lambda nil (interactive)
(kill-new (car (org-ref-get-bibtex-key-and-file))))
(16777318 lambda nil (interactive)
(save-excursion (org-ref-open-citation-at-point)
(kill-new (org-ref-format-bibtex-entry-at-point)))
)
(16777319 . org-ref-google-scholar-at-point)
(16777317 lambda nil "Email entry at point" (interactive)
(org-ref-open-citation-at-point) (org-ref-email-bibtex-entry))
(16777315 . org-ref-wos-citing-at-point)
(16777330 . org-ref-wos-related-at-point)
(16777326 . org-ref-open-notes-at-point)
(16777328 . org-ref-open-pdf-at-point)
(16777333 . org-ref-open-url-at-point)
(16777314 . org-ref-open-citation-at-point)
(16777327 . org-ref-cite-hydra/body) (follow-link . mouse-face)
(mouse-3 . org-find-file-at-mouse) (mouse-2 . org-open-at-mouse))
)
("autocites" :follow
(lambda (_) (funcall org-ref-cite-onclick-function nil)) :export
org-ref-format-autocites :complete org-autocites-complete-link
:help-echo
(lambda (window object position)
(when org-ref-show-citation-on-enter
(save-excursion (goto-char position)
(let
((s (org-ref-format-entry (org-ref-get-bibtex-key-under-cursor)))
)
(with-temp-buffer (insert s) (fill-paragraph) (buffer-string)))
)
)
)
:face org-ref-cite-link-face-fn :display full :keymap
(keymap
(tab lambda nil (interactive)
(funcall org-ref-insert-cite-function))
(S-up . org-ref-sort-citation-link)
(S-right lambda nil (interactive) (org-ref-swap-citation-link 1))
(S-left lambda nil (interactive) (org-ref-swap-citation-link -1))
(C-right . org-ref-next-key) (C-left . org-ref-previous-key)
(16777337 lambda nil
"Paste key at point. Assumes the first thing in the killring is a key."
(interactive) (org-ref-insert-key-at-point (car kill-ring)))
(16777303 lambda nil "Copy all the keys at point." (interactive)
(kill-new (org-element-property :path (org-element-context))))
(16777335 lambda nil (interactive)
(kill-new (car (org-ref-get-bibtex-key-and-file))))
(16777318 lambda nil (interactive)
(save-excursion (org-ref-open-citation-at-point)
(kill-new (org-ref-format-bibtex-entry-at-point)))
)
(16777319 . org-ref-google-scholar-at-point)
(16777317 lambda nil "Email entry at point" (interactive)
(org-ref-open-citation-at-point) (org-ref-email-bibtex-entry))
(16777315 . org-ref-wos-citing-at-point)
(16777330 . org-ref-wos-related-at-point)
(16777326 . org-ref-open-notes-at-point)
(16777328 . org-ref-open-pdf-at-point)
(16777333 . org-ref-open-url-at-point)
(16777314 . org-ref-open-citation-at-point)
(16777327 . org-ref-cite-hydra/body) (follow-link . mouse-face)
(mouse-3 . org-find-file-at-mouse) (mouse-2 . org-open-at-mouse))
)
("supercites" :follow
(lambda (_) (funcall org-ref-cite-onclick-function nil)) :export
org-ref-format-supercites :complete org-supercites-complete-link
:help-echo
(lambda (window object position)
(when org-ref-show-citation-on-enter
(save-excursion (goto-char position)
(let
((s (org-ref-format-entry (org-ref-get-bibtex-key-under-cursor)))
)
(with-temp-buffer (insert s) (fill-paragraph) (buffer-string)))
)
)
)
:face org-ref-cite-link-face-fn :display full :keymap
(keymap
(tab lambda nil (interactive)
(funcall org-ref-insert-cite-function))
(S-up . org-ref-sort-citation-link)
(S-right lambda nil (interactive) (org-ref-swap-citation-link 1))
(S-left lambda nil (interactive) (org-ref-swap-citation-link -1))
(C-right . org-ref-next-key) (C-left . org-ref-previous-key)
(16777337 lambda nil
"Paste key at point. Assumes the first thing in the killring is a key."
(interactive) (org-ref-insert-key-at-point (car kill-ring)))
(16777303 lambda nil "Copy all the keys at point." (interactive)
(kill-new (org-element-property :path (org-element-context))))
(16777335 lambda nil (interactive)
(kill-new (car (org-ref-get-bibtex-key-and-file))))
(16777318 lambda nil (interactive)
(save-excursion (org-ref-open-citation-at-point)
(kill-new (org-ref-format-bibtex-entry-at-point)))
)
(16777319 . org-ref-google-scholar-at-point)
(16777317 lambda nil "Email entry at point" (interactive)
(org-ref-open-citation-at-point) (org-ref-email-bibtex-entry))
(16777315 . org-ref-wos-citing-at-point)
(16777330 . org-ref-wos-related-at-point)
(16777326 . org-ref-open-notes-at-point)
(16777328 . org-ref-open-pdf-at-point)
(16777333 . org-ref-open-url-at-point)
(16777314 . org-ref-open-citation-at-point)
(16777327 . org-ref-cite-hydra/body) (follow-link . mouse-face)
(mouse-3 . org-find-file-at-mouse) (mouse-2 . org-open-at-mouse))
)
("Textcites" :follow
(lambda (_) (funcall org-ref-cite-onclick-function nil)) :export
org-ref-format-Textcites :complete org-Textcites-complete-link
:help-echo
(lambda (window object position)
(when org-ref-show-citation-on-enter
(save-excursion (goto-char position)
(let
((s (org-ref-format-entry (org-ref-get-bibtex-key-under-cursor)))
)
(with-temp-buffer (insert s) (fill-paragraph) (buffer-string)))
)
)
)
:face org-ref-cite-link-face-fn :display full :keymap
(keymap
(tab lambda nil (interactive)
(funcall org-ref-insert-cite-function))
(S-up . org-ref-sort-citation-link)
(S-right lambda nil (interactive) (org-ref-swap-citation-link 1))
(S-left lambda nil (interactive) (org-ref-swap-citation-link -1))
(C-right . org-ref-next-key) (C-left . org-ref-previous-key)
(16777337 lambda nil
"Paste key at point. Assumes the first thing in the killring is a key."
(interactive) (org-ref-insert-key-at-point (car kill-ring)))
(16777303 lambda nil "Copy all the keys at point." (interactive)
(kill-new (org-element-property :path (org-element-context))))
(16777335 lambda nil (interactive)
(kill-new (car (org-ref-get-bibtex-key-and-file))))
(16777318 lambda nil (interactive)
(save-excursion (org-ref-open-citation-at-point)
(kill-new (org-ref-format-bibtex-entry-at-point)))
)
(16777319 . org-ref-google-scholar-at-point)
(16777317 lambda nil "Email entry at point" (interactive)
(org-ref-open-citation-at-point) (org-ref-email-bibtex-entry))
(16777315 . org-ref-wos-citing-at-point)
(16777330 . org-ref-wos-related-at-point)
(16777326 . org-ref-open-notes-at-point)
(16777328 . org-ref-open-pdf-at-point)
(16777333 . org-ref-open-url-at-point)
(16777314 . org-ref-open-citation-at-point)
(16777327 . org-ref-cite-hydra/body) (follow-link . mouse-face)
(mouse-3 . org-find-file-at-mouse) (mouse-2 . org-open-at-mouse))
)
("textcites" :follow
(lambda (_) (funcall org-ref-cite-onclick-function nil)) :export
org-ref-format-textcites :complete org-textcites-complete-link
:help-echo
(lambda (window object position)
(when org-ref-show-citation-on-enter
(save-excursion (goto-char position)
(let
((s (org-ref-format-entry (org-ref-get-bibtex-key-under-cursor)))
)
(with-temp-buffer (insert s) (fill-paragraph) (buffer-string)))
)
)
)
:face org-ref-cite-link-face-fn :display full :keymap
(keymap
(tab lambda nil (interactive)
(funcall org-ref-insert-cite-function))
(S-up . org-ref-sort-citation-link)
(S-right lambda nil (interactive) (org-ref-swap-citation-link 1))
(S-left lambda nil (interactive) (org-ref-swap-citation-link -1))
(C-right . org-ref-next-key) (C-left . org-ref-previous-key)
(16777337 lambda nil
"Paste key at point. Assumes the first thing in the killring is a key."
(interactive) (org-ref-insert-key-at-point (car kill-ring)))
(16777303 lambda nil "Copy all the keys at point." (interactive)
(kill-new (org-element-property :path (org-element-context))))
(16777335 lambda nil (interactive)
(kill-new (car (org-ref-get-bibtex-key-and-file))))
(16777318 lambda nil (interactive)
(save-excursion (org-ref-open-citation-at-point)
(kill-new (org-ref-format-bibtex-entry-at-point)))
)
(16777319 . org-ref-google-scholar-at-point)
(16777317 lambda nil "Email entry at point" (interactive)
(org-ref-open-citation-at-point) (org-ref-email-bibtex-entry))
(16777315 . org-ref-wos-citing-at-point)
(16777330 . org-ref-wos-related-at-point)
(16777326 . org-ref-open-notes-at-point)
(16777328 . org-ref-open-pdf-at-point)
(16777333 . org-ref-open-url-at-point)
(16777314 . org-ref-open-citation-at-point)
(16777327 . org-ref-cite-hydra/body) (follow-link . mouse-face)
(mouse-3 . org-find-file-at-mouse) (mouse-2 . org-open-at-mouse))
)
("Smartcites" :follow
(lambda (_) (funcall org-ref-cite-onclick-function nil)) :export
org-ref-format-Smartcites :complete org-Smartcites-complete-link
:help-echo
(lambda (window object position)
(when org-ref-show-citation-on-enter
(save-excursion (goto-char position)
(let
((s (org-ref-format-entry (org-ref-get-bibtex-key-under-cursor)))
)
(with-temp-buffer (insert s) (fill-paragraph) (buffer-string)))
)
)
)
:face org-ref-cite-link-face-fn :display full :keymap
(keymap
(tab lambda nil (interactive)
(funcall org-ref-insert-cite-function))
(S-up . org-ref-sort-citation-link)
(S-right lambda nil (interactive) (org-ref-swap-citation-link 1))
(S-left lambda nil (interactive) (org-ref-swap-citation-link -1))
(C-right . org-ref-next-key) (C-left . org-ref-previous-key)
(16777337 lambda nil
"Paste key at point. Assumes the first thing in the killring is a key."
(interactive) (org-ref-insert-key-at-point (car kill-ring)))
(16777303 lambda nil "Copy all the keys at point." (interactive)
(kill-new (org-element-property :path (org-element-context))))
(16777335 lambda nil (interactive)
(kill-new (car (org-ref-get-bibtex-key-and-file))))
(16777318 lambda nil (interactive)
(save-excursion (org-ref-open-citation-at-point)
(kill-new (org-ref-format-bibtex-entry-at-point)))
)
(16777319 . org-ref-google-scholar-at-point)
(16777317 lambda nil "Email entry at point" (interactive)
(org-ref-open-citation-at-point) (org-ref-email-bibtex-entry))
(16777315 . org-ref-wos-citing-at-point)
(16777330 . org-ref-wos-related-at-point)
(16777326 . org-ref-open-notes-at-point)
(16777328 . org-ref-open-pdf-at-point)
(16777333 . org-ref-open-url-at-point)
(16777314 . org-ref-open-citation-at-point)
(16777327 . org-ref-cite-hydra/body) (follow-link . mouse-face)
(mouse-3 . org-find-file-at-mouse) (mouse-2 . org-open-at-mouse))
)
("smartcites" :follow
(lambda (_) (funcall org-ref-cite-onclick-function nil)) :export
org-ref-format-smartcites :complete org-smartcites-complete-link
:help-echo
(lambda (window object position)
(when org-ref-show-citation-on-enter
(save-excursion (goto-char position)
(let
((s (org-ref-format-entry (org-ref-get-bibtex-key-under-cursor)))
)
(with-temp-buffer (insert s) (fill-paragraph) (buffer-string)))
)
)
)
:face org-ref-cite-link-face-fn :display full :keymap
(keymap
(tab lambda nil (interactive)
(funcall org-ref-insert-cite-function))
(S-up . org-ref-sort-citation-link)
(S-right lambda nil (interactive) (org-ref-swap-citation-link 1))
(S-left lambda nil (interactive) (org-ref-swap-citation-link -1))
(C-right . org-ref-next-key) (C-left . org-ref-previous-key)
(16777337 lambda nil
"Paste key at point. Assumes the first thing in the killring is a key."
(interactive) (org-ref-insert-key-at-point (car kill-ring)))
(16777303 lambda nil "Copy all the keys at point." (interactive)
(kill-new (org-element-property :path (org-element-context))))
(16777335 lambda nil (interactive)
(kill-new (car (org-ref-get-bibtex-key-and-file))))
(16777318 lambda nil (interactive)
(save-excursion (org-ref-open-citation-at-point)
(kill-new (org-ref-format-bibtex-entry-at-point)))
)
(16777319 . org-ref-google-scholar-at-point)
(16777317 lambda nil "Email entry at point" (interactive)
(org-ref-open-citation-at-point) (org-ref-email-bibtex-entry))
(16777315 . org-ref-wos-citing-at-point)
(16777330 . org-ref-wos-related-at-point)
(16777326 . org-ref-open-notes-at-point)
(16777328 . org-ref-open-pdf-at-point)
(16777333 . org-ref-open-url-at-point)
(16777314 . org-ref-open-citation-at-point)
(16777327 . org-ref-cite-hydra/body) (follow-link . mouse-face)
(mouse-3 . org-find-file-at-mouse) (mouse-2 . org-open-at-mouse))
)
("footcitetexts" :follow
(lambda (_) (funcall org-ref-cite-onclick-function nil)) :export
org-ref-format-footcitetexts :complete
org-footcitetexts-complete-link :help-echo
(lambda (window object position)
(when org-ref-show-citation-on-enter
(save-excursion (goto-char position)
(let
((s (org-ref-format-entry (org-ref-get-bibtex-key-under-cursor)))
)
(with-temp-buffer (insert s) (fill-paragraph) (buffer-string)))
)
)
)
:face org-ref-cite-link-face-fn :display full :keymap
(keymap
(tab lambda nil (interactive)
(funcall org-ref-insert-cite-function))
(S-up . org-ref-sort-citation-link)
(S-right lambda nil (interactive) (org-ref-swap-citation-link 1))
(S-left lambda nil (interactive) (org-ref-swap-citation-link -1))
(C-right . org-ref-next-key) (C-left . org-ref-previous-key)
(16777337 lambda nil
"Paste key at point. Assumes the first thing in the killring is a key."
(interactive) (org-ref-insert-key-at-point (car kill-ring)))
(16777303 lambda nil "Copy all the keys at point." (interactive)
(kill-new (org-element-property :path (org-element-context))))
(16777335 lambda nil (interactive)
(kill-new (car (org-ref-get-bibtex-key-and-file))))
(16777318 lambda nil (interactive)
(save-excursion (org-ref-open-citation-at-point)
(kill-new (org-ref-format-bibtex-entry-at-point)))
)
(16777319 . org-ref-google-scholar-at-point)
(16777317 lambda nil "Email entry at point" (interactive)
(org-ref-open-citation-at-point) (org-ref-email-bibtex-entry))
(16777315 . org-ref-wos-citing-at-point)
(16777330 . org-ref-wos-related-at-point)
(16777326 . org-ref-open-notes-at-point)
(16777328 . org-ref-open-pdf-at-point)
(16777333 . org-ref-open-url-at-point)
(16777314 . org-ref-open-citation-at-point)
(16777327 . org-ref-cite-hydra/body) (follow-link . mouse-face)
(mouse-3 . org-find-file-at-mouse) (mouse-2 . org-open-at-mouse))
)
("footcites" :follow
(lambda (_) (funcall org-ref-cite-onclick-function nil)) :export
org-ref-format-footcites :complete org-footcites-complete-link
:help-echo
(lambda (window object position)
(when org-ref-show-citation-on-enter
(save-excursion (goto-char position)
(let
((s (org-ref-format-entry (org-ref-get-bibtex-key-under-cursor)))
)
(with-temp-buffer (insert s) (fill-paragraph) (buffer-string)))
)
)
)
:face org-ref-cite-link-face-fn :display full :keymap
(keymap
(tab lambda nil (interactive)
(funcall org-ref-insert-cite-function))
(S-up . org-ref-sort-citation-link)
(S-right lambda nil (interactive) (org-ref-swap-citation-link 1))
(S-left lambda nil (interactive) (org-ref-swap-citation-link -1))
(C-right . org-ref-next-key) (C-left . org-ref-previous-key)
(16777337 lambda nil
"Paste key at point. Assumes the first thing in the killring is a key."
(interactive) (org-ref-insert-key-at-point (car kill-ring)))
(16777303 lambda nil "Copy all the keys at point." (interactive)
(kill-new (org-element-property :path (org-element-context))))
(16777335 lambda nil (interactive)
(kill-new (car (org-ref-get-bibtex-key-and-file))))
(16777318 lambda nil (interactive)
(save-excursion (org-ref-open-citation-at-point)
(kill-new (org-ref-format-bibtex-entry-at-point)))
)
(16777319 . org-ref-google-scholar-at-point)
(16777317 lambda nil "Email entry at point" (interactive)
(org-ref-open-citation-at-point) (org-ref-email-bibtex-entry))
(16777315 . org-ref-wos-citing-at-point)
(16777330 . org-ref-wos-related-at-point)
(16777326 . org-ref-open-notes-at-point)
(16777328 . org-ref-open-pdf-at-point)
(16777333 . org-ref-open-url-at-point)
(16777314 . org-ref-open-citation-at-point)
(16777327 . org-ref-cite-hydra/body) (follow-link . mouse-face)
(mouse-3 . org-find-file-at-mouse) (mouse-2 . org-open-at-mouse))
)
("Parencites" :follow
(lambda (_) (funcall org-ref-cite-onclick-function nil)) :export
org-ref-format-Parencites :complete org-Parencites-complete-link
:help-echo
(lambda (window object position)
(when org-ref-show-citation-on-enter
(save-excursion (goto-char position)
(let
((s (org-ref-format-entry (org-ref-get-bibtex-key-under-cursor)))
)
(with-temp-buffer (insert s) (fill-paragraph) (buffer-string)))
)
)
)
:face org-ref-cite-link-face-fn :display full :keymap
(keymap
(tab lambda nil (interactive)
(funcall org-ref-insert-cite-function))
(S-up . org-ref-sort-citation-link)
(S-right lambda nil (interactive) (org-ref-swap-citation-link 1))
(S-left lambda nil (interactive) (org-ref-swap-citation-link -1))
(C-right . org-ref-next-key) (C-left . org-ref-previous-key)
(16777337 lambda nil
"Paste key at point. Assumes the first thing in the killring is a key."
(interactive) (org-ref-insert-key-at-point (car kill-ring)))
(16777303 lambda nil "Copy all the keys at point." (interactive)
(kill-new (org-element-property :path (org-element-context))))
(16777335 lambda nil (interactive)
(kill-new (car (org-ref-get-bibtex-key-and-file))))
(16777318 lambda nil (interactive)
(save-excursion (org-ref-open-citation-at-point)
(kill-new (org-ref-format-bibtex-entry-at-point)))
)
(16777319 . org-ref-google-scholar-at-point)
(16777317 lambda nil "Email entry at point" (interactive)
(org-ref-open-citation-at-point) (org-ref-email-bibtex-entry))
(16777315 . org-ref-wos-citing-at-point)
(16777330 . org-ref-wos-related-at-point)
(16777326 . org-ref-open-notes-at-point)
(16777328 . org-ref-open-pdf-at-point)
(16777333 . org-ref-open-url-at-point)
(16777314 . org-ref-open-citation-at-point)
(16777327 . org-ref-cite-hydra/body) (follow-link . mouse-face)
(mouse-3 . org-find-file-at-mouse) (mouse-2 . org-open-at-mouse))
)
("parencites" :follow
(lambda (_) (funcall org-ref-cite-onclick-function nil)) :export
org-ref-format-parencites :complete org-parencites-complete-link
:help-echo
(lambda (window object position)
(when org-ref-show-citation-on-enter
(save-excursion (goto-char position)
(let
((s (org-ref-format-entry (org-ref-get-bibtex-key-under-cursor)))
)
(with-temp-buffer (insert s) (fill-paragraph) (buffer-string)))
)
)
)
:face org-ref-cite-link-face-fn :display full :keymap
(keymap
(tab lambda nil (interactive)
(funcall org-ref-insert-cite-function))
(S-up . org-ref-sort-citation-link)
(S-right lambda nil (interactive) (org-ref-swap-citation-link 1))
(S-left lambda nil (interactive) (org-ref-swap-citation-link -1))
(C-right . org-ref-next-key) (C-left . org-ref-previous-key)
(16777337 lambda nil
"Paste key at point. Assumes the first thing in the killring is a key."
(interactive) (org-ref-insert-key-at-point (car kill-ring)))
(16777303 lambda nil "Copy all the keys at point." (interactive)
(kill-new (org-element-property :path (org-element-context))))
(16777335 lambda nil (interactive)
(kill-new (car (org-ref-get-bibtex-key-and-file))))
(16777318 lambda nil (interactive)
(save-excursion (org-ref-open-citation-at-point)
(kill-new (org-ref-format-bibtex-entry-at-point)))
)
(16777319 . org-ref-google-scholar-at-point)
(16777317 lambda nil "Email entry at point" (interactive)
(org-ref-open-citation-at-point) (org-ref-email-bibtex-entry))
(16777315 . org-ref-wos-citing-at-point)
(16777330 . org-ref-wos-related-at-point)
(16777326 . org-ref-open-notes-at-point)
(16777328 . org-ref-open-pdf-at-point)
(16777333 . org-ref-open-url-at-point)
(16777314 . org-ref-open-citation-at-point)
(16777327 . org-ref-cite-hydra/body) (follow-link . mouse-face)
(mouse-3 . org-find-file-at-mouse) (mouse-2 . org-open-at-mouse))
)
("Cites" :follow
(lambda (_) (funcall org-ref-cite-onclick-function nil)) :export
org-ref-format-Cites :complete org-Cites-complete-link :help-echo
(lambda (window object position)
(when org-ref-show-citation-on-enter
(save-excursion (goto-char position)
(let
((s (org-ref-format-entry (org-ref-get-bibtex-key-under-cursor)))
)
(with-temp-buffer (insert s) (fill-paragraph) (buffer-string)))
)
)
)
:face org-ref-cite-link-face-fn :display full :keymap
(keymap
(tab lambda nil (interactive)
(funcall org-ref-insert-cite-function))
(S-up . org-ref-sort-citation-link)
(S-right lambda nil (interactive) (org-ref-swap-citation-link 1))
(S-left lambda nil (interactive) (org-ref-swap-citation-link -1))
(C-right . org-ref-next-key) (C-left . org-ref-previous-key)
(16777337 lambda nil
"Paste key at point. Assumes the first thing in the killring is a key."
(interactive) (org-ref-insert-key-at-point (car kill-ring)))
(16777303 lambda nil "Copy all the keys at point." (interactive)
(kill-new (org-element-property :path (org-element-context))))
(16777335 lambda nil (interactive)
(kill-new (car (org-ref-get-bibtex-key-and-file))))
(16777318 lambda nil (interactive)
(save-excursion (org-ref-open-citation-at-point)
(kill-new (org-ref-format-bibtex-entry-at-point)))
)
(16777319 . org-ref-google-scholar-at-point)
(16777317 lambda nil "Email entry at point" (interactive)
(org-ref-open-citation-at-point) (org-ref-email-bibtex-entry))
(16777315 . org-ref-wos-citing-at-point)
(16777330 . org-ref-wos-related-at-point)
(16777326 . org-ref-open-notes-at-point)
(16777328 . org-ref-open-pdf-at-point)
(16777333 . org-ref-open-url-at-point)
(16777314 . org-ref-open-citation-at-point)
(16777327 . org-ref-cite-hydra/body) (follow-link . mouse-face)
(mouse-3 . org-find-file-at-mouse) (mouse-2 . org-open-at-mouse))
)
("cites" :follow
(lambda (_) (funcall org-ref-cite-onclick-function nil)) :export
org-ref-format-cites :complete org-cites-complete-link :help-echo
(lambda (window object position)
(when org-ref-show-citation-on-enter
(save-excursion (goto-char position)
(let
((s (org-ref-format-entry (org-ref-get-bibtex-key-under-cursor)))
)
(with-temp-buffer (insert s) (fill-paragraph) (buffer-string)))
)
)
)
:face org-ref-cite-link-face-fn :display full :keymap
(keymap
(tab lambda nil (interactive)
(funcall org-ref-insert-cite-function))
(S-up . org-ref-sort-citation-link)
(S-right lambda nil (interactive) (org-ref-swap-citation-link 1))
(S-left lambda nil (interactive) (org-ref-swap-citation-link -1))
(C-right . org-ref-next-key) (C-left . org-ref-previous-key)
(16777337 lambda nil
"Paste key at point. Assumes the first thing in the killring is a key."
(interactive) (org-ref-insert-key-at-point (car kill-ring)))
(16777303 lambda nil "Copy all the keys at point." (interactive)
(kill-new (org-element-property :path (org-element-context))))
(16777335 lambda nil (interactive)
(kill-new (car (org-ref-get-bibtex-key-and-file))))
(16777318 lambda nil (interactive)
(save-excursion (org-ref-open-citation-at-point)
(kill-new (org-ref-format-bibtex-entry-at-point)))
)
(16777319 . org-ref-google-scholar-at-point)
(16777317 lambda nil "Email entry at point" (interactive)
(org-ref-open-citation-at-point) (org-ref-email-bibtex-entry))
(16777315 . org-ref-wos-citing-at-point)
(16777330 . org-ref-wos-related-at-point)
(16777326 . org-ref-open-notes-at-point)
(16777328 . org-ref-open-pdf-at-point)
(16777333 . org-ref-open-url-at-point)
(16777314 . org-ref-open-citation-at-point)
(16777327 . org-ref-cite-hydra/body) (follow-link . mouse-face)
(mouse-3 . org-find-file-at-mouse) (mouse-2 . org-open-at-mouse))
)
("fnotecite" :follow
(lambda (_) (funcall org-ref-cite-onclick-function nil)) :export
org-ref-format-fnotecite :complete org-fnotecite-complete-link
:help-echo
(lambda (window object position)
(when org-ref-show-citation-on-enter
(save-excursion (goto-char position)
(let
((s (org-ref-format-entry (org-ref-get-bibtex-key-under-cursor)))
)
(with-temp-buffer (insert s) (fill-paragraph) (buffer-string)))
)
)
)
:face org-ref-cite-link-face-fn :display full :keymap
(keymap
(tab lambda nil (interactive)
(funcall org-ref-insert-cite-function))
(S-up . org-ref-sort-citation-link)
(S-right lambda nil (interactive) (org-ref-swap-citation-link 1))
(S-left lambda nil (interactive) (org-ref-swap-citation-link -1))
(C-right . org-ref-next-key) (C-left . org-ref-previous-key)
(16777337 lambda nil
"Paste key at point. Assumes the first thing in the killring is a key."
(interactive) (org-ref-insert-key-at-point (car kill-ring)))
(16777303 lambda nil "Copy all the keys at point." (interactive)
(kill-new (org-element-property :path (org-element-context))))
(16777335 lambda nil (interactive)
(kill-new (car (org-ref-get-bibtex-key-and-file))))
(16777318 lambda nil (interactive)
(save-excursion (org-ref-open-citation-at-point)
(kill-new (org-ref-format-bibtex-entry-at-point)))
)
(16777319 . org-ref-google-scholar-at-point)
(16777317 lambda nil "Email entry at point" (interactive)
(org-ref-open-citation-at-point) (org-ref-email-bibtex-entry))
(16777315 . org-ref-wos-citing-at-point)
(16777330 . org-ref-wos-related-at-point)
(16777326 . org-ref-open-notes-at-point)
(16777328 . org-ref-open-pdf-at-point)
(16777333 . org-ref-open-url-at-point)
(16777314 . org-ref-open-citation-at-point)
(16777327 . org-ref-cite-hydra/body) (follow-link . mouse-face)
(mouse-3 . org-find-file-at-mouse) (mouse-2 . org-open-at-mouse))
)
("Pnotecite" :follow
(lambda (_) (funcall org-ref-cite-onclick-function nil)) :export
org-ref-format-Pnotecite :complete org-Pnotecite-complete-link
:help-echo
(lambda (window object position)
(when org-ref-show-citation-on-enter
(save-excursion (goto-char position)
(let
((s (org-ref-format-entry (org-ref-get-bibtex-key-under-cursor)))
)
(with-temp-buffer (insert s) (fill-paragraph) (buffer-string)))
)
)
)
:face org-ref-cite-link-face-fn :display full :keymap
(keymap
(tab lambda nil (interactive)
(funcall org-ref-insert-cite-function))
(S-up . org-ref-sort-citation-link)
(S-right lambda nil (interactive) (org-ref-swap-citation-link 1))
(S-left lambda nil (interactive) (org-ref-swap-citation-link -1))
(C-right . org-ref-next-key) (C-left . org-ref-previous-key)
(16777337 lambda nil
"Paste key at point. Assumes the first thing in the killring is a key."
(interactive) (org-ref-insert-key-at-point (car kill-ring)))
(16777303 lambda nil "Copy all the keys at point." (interactive)
(kill-new (org-element-property :path (org-element-context))))
(16777335 lambda nil (interactive)
(kill-new (car (org-ref-get-bibtex-key-and-file))))
(16777318 lambda nil (interactive)
(save-excursion (org-ref-open-citation-at-point)
(kill-new (org-ref-format-bibtex-entry-at-point)))
)
(16777319 . org-ref-google-scholar-at-point)
(16777317 lambda nil "Email entry at point" (interactive)
(org-ref-open-citation-at-point) (org-ref-email-bibtex-entry))
(16777315 . org-ref-wos-citing-at-point)
(16777330 . org-ref-wos-related-at-point)
(16777326 . org-ref-open-notes-at-point)
(16777328 . org-ref-open-pdf-at-point)
(16777333 . org-ref-open-url-at-point)
(16777314 . org-ref-open-citation-at-point)
(16777327 . org-ref-cite-hydra/body) (follow-link . mouse-face)
(mouse-3 . org-find-file-at-mouse) (mouse-2 . org-open-at-mouse))
)
("pnotecite" :follow
(lambda (_) (funcall org-ref-cite-onclick-function nil)) :export
org-ref-format-pnotecite :complete org-pnotecite-complete-link
:help-echo
(lambda (window object position)
(when org-ref-show-citation-on-enter
(save-excursion (goto-char position)
(let
((s (org-ref-format-entry (org-ref-get-bibtex-key-under-cursor)))
)
(with-temp-buffer (insert s) (fill-paragraph) (buffer-string)))
)
)
)
:face org-ref-cite-link-face-fn :display full :keymap
(keymap
(tab lambda nil (interactive)
(funcall org-ref-insert-cite-function))
(S-up . org-ref-sort-citation-link)
(S-right lambda nil (interactive) (org-ref-swap-citation-link 1))
(S-left lambda nil (interactive) (org-ref-swap-citation-link -1))
(C-right . org-ref-next-key) (C-left . org-ref-previous-key)
(16777337 lambda nil
"Paste key at point. Assumes the first thing in the killring is a key."
(interactive) (org-ref-insert-key-at-point (car kill-ring)))
(16777303 lambda nil "Copy all the keys at point." (interactive)
(kill-new (org-element-property :path (org-element-context))))
(16777335 lambda nil (interactive)
(kill-new (car (org-ref-get-bibtex-key-and-file))))
(16777318 lambda nil (interactive)
(save-excursion (org-ref-open-citation-at-point)
(kill-new (org-ref-format-bibtex-entry-at-point)))
)
(16777319 . org-ref-google-scholar-at-point)
(16777317 lambda nil "Email entry at point" (interactive)
(org-ref-open-citation-at-point) (org-ref-email-bibtex-entry))
(16777315 . org-ref-wos-citing-at-point)
(16777330 . org-ref-wos-related-at-point)
(16777326 . org-ref-open-notes-at-point)
(16777328 . org-ref-open-pdf-at-point)
(16777333 . org-ref-open-url-at-point)
(16777314 . org-ref-open-citation-at-point)
(16777327 . org-ref-cite-hydra/body) (follow-link . mouse-face)
(mouse-3 . org-find-file-at-mouse) (mouse-2 . org-open-at-mouse))
)
("Notecite" :follow
(lambda (_) (funcall org-ref-cite-onclick-function nil)) :export
org-ref-format-Notecite :complete org-Notecite-complete-link
:help-echo
(lambda (window object position)
(when org-ref-show-citation-on-enter
(save-excursion (goto-char position)
(let
((s (org-ref-format-entry (org-ref-get-bibtex-key-under-cursor)))
)
(with-temp-buffer (insert s) (fill-paragraph) (buffer-string)))
)
)
)
:face org-ref-cite-link-face-fn :display full :keymap
(keymap
(tab lambda nil (interactive)
(funcall org-ref-insert-cite-function))
(S-up . org-ref-sort-citation-link)
(S-right lambda nil (interactive) (org-ref-swap-citation-link 1))
(S-left lambda nil (interactive) (org-ref-swap-citation-link -1))
(C-right . org-ref-next-key) (C-left . org-ref-previous-key)
(16777337 lambda nil
"Paste key at point. Assumes the first thing in the killring is a key."
(interactive) (org-ref-insert-key-at-point (car kill-ring)))
(16777303 lambda nil "Copy all the keys at point." (interactive)
(kill-new (org-element-property :path (org-element-context))))
(16777335 lambda nil (interactive)
(kill-new (car (org-ref-get-bibtex-key-and-file))))
(16777318 lambda nil (interactive)
(save-excursion (org-ref-open-citation-at-point)
(kill-new (org-ref-format-bibtex-entry-at-point)))
)
(16777319 . org-ref-google-scholar-at-point)
(16777317 lambda nil "Email entry at point" (interactive)
(org-ref-open-citation-at-point) (org-ref-email-bibtex-entry))
(16777315 . org-ref-wos-citing-at-point)
(16777330 . org-ref-wos-related-at-point)
(16777326 . org-ref-open-notes-at-point)
(16777328 . org-ref-open-pdf-at-point)
(16777333 . org-ref-open-url-at-point)
(16777314 . org-ref-open-citation-at-point)
(16777327 . org-ref-cite-hydra/body) (follow-link . mouse-face)
(mouse-3 . org-find-file-at-mouse) (mouse-2 . org-open-at-mouse))
)
("notecite" :follow
(lambda (_) (funcall org-ref-cite-onclick-function nil)) :export
org-ref-format-notecite :complete org-notecite-complete-link
:help-echo
(lambda (window object position)
(when org-ref-show-citation-on-enter
(save-excursion (goto-char position)
(let
((s (org-ref-format-entry (org-ref-get-bibtex-key-under-cursor)))
)
(with-temp-buffer (insert s) (fill-paragraph) (buffer-string)))
)
)
)
:face org-ref-cite-link-face-fn :display full :keymap
(keymap
(tab lambda nil (interactive)
(funcall org-ref-insert-cite-function))
(S-up . org-ref-sort-citation-link)
(S-right lambda nil (interactive) (org-ref-swap-citation-link 1))
(S-left lambda nil (interactive) (org-ref-swap-citation-link -1))
(C-right . org-ref-next-key) (C-left . org-ref-previous-key)
(16777337 lambda nil
"Paste key at point. Assumes the first thing in the killring is a key."
(interactive) (org-ref-insert-key-at-point (car kill-ring)))
(16777303 lambda nil "Copy all the keys at point." (interactive)
(kill-new (org-element-property :path (org-element-context))))
(16777335 lambda nil (interactive)
(kill-new (car (org-ref-get-bibtex-key-and-file))))
(16777318 lambda nil (interactive)
(save-excursion (org-ref-open-citation-at-point)
(kill-new (org-ref-format-bibtex-entry-at-point)))
)
(16777319 . org-ref-google-scholar-at-point)
(16777317 lambda nil "Email entry at point" (interactive)
(org-ref-open-citation-at-point) (org-ref-email-bibtex-entry))
(16777315 . org-ref-wos-citing-at-point)
(16777330 . org-ref-wos-related-at-point)
(16777326 . org-ref-open-notes-at-point)
(16777328 . org-ref-open-pdf-at-point)
(16777333 . org-ref-open-url-at-point)
(16777314 . org-ref-open-citation-at-point)
(16777327 . org-ref-cite-hydra/body) (follow-link . mouse-face)
(mouse-3 . org-find-file-at-mouse) (mouse-2 . org-open-at-mouse))
)
("footfullcite" :follow
(lambda (_) (funcall org-ref-cite-onclick-function nil)) :export
org-ref-format-footfullcite :complete org-footfullcite-complete-link
:help-echo
(lambda (window object position)
(when org-ref-show-citation-on-enter
(save-excursion (goto-char position)
(let
((s (org-ref-format-entry (org-ref-get-bibtex-key-under-cursor)))
)
(with-temp-buffer (insert s) (fill-paragraph) (buffer-string)))
)
)
)
:face org-ref-cite-link-face-fn :display full :keymap
(keymap
(tab lambda nil (interactive)
(funcall org-ref-insert-cite-function))
(S-up . org-ref-sort-citation-link)
(S-right lambda nil (interactive) (org-ref-swap-citation-link 1))
(S-left lambda nil (interactive) (org-ref-swap-citation-link -1))
(C-right . org-ref-next-key) (C-left . org-ref-previous-key)
(16777337 lambda nil
"Paste key at point. Assumes the first thing in the killring is a key."
(interactive) (org-ref-insert-key-at-point (car kill-ring)))
(16777303 lambda nil "Copy all the keys at point." (interactive)
(kill-new (org-element-property :path (org-element-context))))
(16777335 lambda nil (interactive)
(kill-new (car (org-ref-get-bibtex-key-and-file))))
(16777318 lambda nil (interactive)
(save-excursion (org-ref-open-citation-at-point)
(kill-new (org-ref-format-bibtex-entry-at-point)))
)
(16777319 . org-ref-google-scholar-at-point)
(16777317 lambda nil "Email entry at point" (interactive)
(org-ref-open-citation-at-point) (org-ref-email-bibtex-entry))
(16777315 . org-ref-wos-citing-at-point)
(16777330 . org-ref-wos-related-at-point)
(16777326 . org-ref-open-notes-at-point)
(16777328 . org-ref-open-pdf-at-point)
(16777333 . org-ref-open-url-at-point)
(16777314 . org-ref-open-citation-at-point)
(16777327 . org-ref-cite-hydra/body) (follow-link . mouse-face)
(mouse-3 . org-find-file-at-mouse) (mouse-2 . org-open-at-mouse))
)
("fullcite" :follow
(lambda (_) (funcall org-ref-cite-onclick-function nil)) :export
org-ref-format-fullcite :complete org-fullcite-complete-link
:help-echo
(lambda (window object position)
(when org-ref-show-citation-on-enter
(save-excursion (goto-char position)
(let
((s (org-ref-format-entry (org-ref-get-bibtex-key-under-cursor)))
)
(with-temp-buffer (insert s) (fill-paragraph) (buffer-string)))
)
)
)
:face org-ref-cite-link-face-fn :display full :keymap
(keymap
(tab lambda nil (interactive)
(funcall org-ref-insert-cite-function))
(S-up . org-ref-sort-citation-link)
(S-right lambda nil (interactive) (org-ref-swap-citation-link 1))
(S-left lambda nil (interactive) (org-ref-swap-citation-link -1))
(C-right . org-ref-next-key) (C-left . org-ref-previous-key)
(16777337 lambda nil
"Paste key at point. Assumes the first thing in the killring is a key."
(interactive) (org-ref-insert-key-at-point (car kill-ring)))
(16777303 lambda nil "Copy all the keys at point." (interactive)
(kill-new (org-element-property :path (org-element-context))))
(16777335 lambda nil (interactive)
(kill-new (car (org-ref-get-bibtex-key-and-file))))
(16777318 lambda nil (interactive)
(save-excursion (org-ref-open-citation-at-point)
(kill-new (org-ref-format-bibtex-entry-at-point)))
)
(16777319 . org-ref-google-scholar-at-point)
(16777317 lambda nil "Email entry at point" (interactive)
(org-ref-open-citation-at-point) (org-ref-email-bibtex-entry))
(16777315 . org-ref-wos-citing-at-point)
(16777330 . org-ref-wos-related-at-point)
(16777326 . org-ref-open-notes-at-point)
(16777328 . org-ref-open-pdf-at-point)
(16777333 . org-ref-open-url-at-point)
(16777314 . org-ref-open-citation-at-point)
(16777327 . org-ref-cite-hydra/body) (follow-link . mouse-face)
(mouse-3 . org-find-file-at-mouse) (mouse-2 . org-open-at-mouse))
)
("citeurl" :follow
(lambda (_) (funcall org-ref-cite-onclick-function nil)) :export
org-ref-format-citeurl :complete org-citeurl-complete-link
:help-echo
(lambda (window object position)
(when org-ref-show-citation-on-enter
(save-excursion (goto-char position)
(let
((s (org-ref-format-entry (org-ref-get-bibtex-key-under-cursor)))
)
(with-temp-buffer (insert s) (fill-paragraph) (buffer-string)))
)
)
)
:face org-ref-cite-link-face-fn :display full :keymap
(keymap
(tab lambda nil (interactive)
(funcall org-ref-insert-cite-function))
(S-up . org-ref-sort-citation-link)
(S-right lambda nil (interactive) (org-ref-swap-citation-link 1))
(S-left lambda nil (interactive) (org-ref-swap-citation-link -1))
(C-right . org-ref-next-key) (C-left . org-ref-previous-key)
(16777337 lambda nil
"Paste key at point. Assumes the first thing in the killring is a key."
(interactive) (org-ref-insert-key-at-point (car kill-ring)))
(16777303 lambda nil "Copy all the keys at point." (interactive)
(kill-new (org-element-property :path (org-element-context))))
(16777335 lambda nil (interactive)
(kill-new (car (org-ref-get-bibtex-key-and-file))))
(16777318 lambda nil (interactive)
(save-excursion (org-ref-open-citation-at-point)
(kill-new (org-ref-format-bibtex-entry-at-point)))
)
(16777319 . org-ref-google-scholar-at-point)
(16777317 lambda nil "Email entry at point" (interactive)
(org-ref-open-citation-at-point) (org-ref-email-bibtex-entry))
(16777315 . org-ref-wos-citing-at-point)
(16777330 . org-ref-wos-related-at-point)
(16777326 . org-ref-open-notes-at-point)
(16777328 . org-ref-open-pdf-at-point)
(16777333 . org-ref-open-url-at-point)
(16777314 . org-ref-open-citation-at-point)
(16777327 . org-ref-cite-hydra/body) (follow-link . mouse-face)
(mouse-3 . org-find-file-at-mouse) (mouse-2 . org-open-at-mouse))
)
("citedate*" :follow
(lambda (_) (funcall org-ref-cite-onclick-function nil)) :export
org-ref-format-citedate* :complete org-citedate*-complete-link
:help-echo
(lambda (window object position)
(when org-ref-show-citation-on-enter
(save-excursion (goto-char position)
(let
((s (org-ref-format-entry (org-ref-get-bibtex-key-under-cursor)))
)
(with-temp-buffer (insert s) (fill-paragraph) (buffer-string)))
)
)
)
:face org-ref-cite-link-face-fn :display full :keymap
(keymap
(tab lambda nil (interactive)
(funcall org-ref-insert-cite-function))
(S-up . org-ref-sort-citation-link)
(S-right lambda nil (interactive) (org-ref-swap-citation-link 1))
(S-left lambda nil (interactive) (org-ref-swap-citation-link -1))
(C-right . org-ref-next-key) (C-left . org-ref-previous-key)
(16777337 lambda nil
"Paste key at point. Assumes the first thing in the killring is a key."
(interactive) (org-ref-insert-key-at-point (car kill-ring)))
(16777303 lambda nil "Copy all the keys at point." (interactive)
(kill-new (org-element-property :path (org-element-context))))
(16777335 lambda nil (interactive)
(kill-new (car (org-ref-get-bibtex-key-and-file))))
(16777318 lambda nil (interactive)
(save-excursion (org-ref-open-citation-at-point)
(kill-new (org-ref-format-bibtex-entry-at-point)))
)
(16777319 . org-ref-google-scholar-at-point)
(16777317 lambda nil "Email entry at point" (interactive)
(org-ref-open-citation-at-point) (org-ref-email-bibtex-entry))
(16777315 . org-ref-wos-citing-at-point)
(16777330 . org-ref-wos-related-at-point)
(16777326 . org-ref-open-notes-at-point)
(16777328 . org-ref-open-pdf-at-point)
(16777333 . org-ref-open-url-at-point)
(16777314 . org-ref-open-citation-at-point)
(16777327 . org-ref-cite-hydra/body) (follow-link . mouse-face)
(mouse-3 . org-find-file-at-mouse) (mouse-2 . org-open-at-mouse))
)
("citedate" :follow
(lambda (_) (funcall org-ref-cite-onclick-function nil)) :export
org-ref-format-citedate :complete org-citedate-complete-link
:help-echo
(lambda (window object position)
(when org-ref-show-citation-on-enter
(save-excursion (goto-char position)
(let
((s (org-ref-format-entry (org-ref-get-bibtex-key-under-cursor)))
)
(with-temp-buffer (insert s) (fill-paragraph) (buffer-string)))
)
)
)
:face org-ref-cite-link-face-fn :display full :keymap
(keymap
(tab lambda nil (interactive)
(funcall org-ref-insert-cite-function))
(S-up . org-ref-sort-citation-link)
(S-right lambda nil (interactive) (org-ref-swap-citation-link 1))
(S-left lambda nil (interactive) (org-ref-swap-citation-link -1))
(C-right . org-ref-next-key) (C-left . org-ref-previous-key)
(16777337 lambda nil
"Paste key at point. Assumes the first thing in the killring is a key."
(interactive) (org-ref-insert-key-at-point (car kill-ring)))
(16777303 lambda nil "Copy all the keys at point." (interactive)
(kill-new (org-element-property :path (org-element-context))))
(16777335 lambda nil (interactive)
(kill-new (car (org-ref-get-bibtex-key-and-file))))
(16777318 lambda nil (interactive)
(save-excursion (org-ref-open-citation-at-point)
(kill-new (org-ref-format-bibtex-entry-at-point)))
)
(16777319 . org-ref-google-scholar-at-point)
(16777317 lambda nil "Email entry at point" (interactive)
(org-ref-open-citation-at-point) (org-ref-email-bibtex-entry))
(16777315 . org-ref-wos-citing-at-point)
(16777330 . org-ref-wos-related-at-point)
(16777326 . org-ref-open-notes-at-point)
(16777328 . org-ref-open-pdf-at-point)
(16777333 . org-ref-open-url-at-point)
(16777314 . org-ref-open-citation-at-point)
(16777327 . org-ref-cite-hydra/body) (follow-link . mouse-face)
(mouse-3 . org-find-file-at-mouse) (mouse-2 . org-open-at-mouse))
)
("citetitle*" :follow
(lambda (_) (funcall org-ref-cite-onclick-function nil)) :export
org-ref-format-citetitle* :complete org-citetitle*-complete-link
:help-echo
(lambda (window object position)
(when org-ref-show-citation-on-enter
(save-excursion (goto-char position)
(let
((s (org-ref-format-entry (org-ref-get-bibtex-key-under-cursor)))
)
(with-temp-buffer (insert s) (fill-paragraph) (buffer-string)))
)
)
)
:face org-ref-cite-link-face-fn :display full :keymap
(keymap
(tab lambda nil (interactive)
(funcall org-ref-insert-cite-function))
(S-up . org-ref-sort-citation-link)
(S-right lambda nil (interactive) (org-ref-swap-citation-link 1))
(S-left lambda nil (interactive) (org-ref-swap-citation-link -1))
(C-right . org-ref-next-key) (C-left . org-ref-previous-key)
(16777337 lambda nil
"Paste key at point. Assumes the first thing in the killring is a key."
(interactive) (org-ref-insert-key-at-point (car kill-ring)))
(16777303 lambda nil "Copy all the keys at point." (interactive)
(kill-new (org-element-property :path (org-element-context))))
(16777335 lambda nil (interactive)
(kill-new (car (org-ref-get-bibtex-key-and-file))))
(16777318 lambda nil (interactive)
(save-excursion (org-ref-open-citation-at-point)
(kill-new (org-ref-format-bibtex-entry-at-point)))
)
(16777319 . org-ref-google-scholar-at-point)
(16777317 lambda nil "Email entry at point" (interactive)
(org-ref-open-citation-at-point) (org-ref-email-bibtex-entry))
(16777315 . org-ref-wos-citing-at-point)
(16777330 . org-ref-wos-related-at-point)
(16777326 . org-ref-open-notes-at-point)
(16777328 . org-ref-open-pdf-at-point)
(16777333 . org-ref-open-url-at-point)
(16777314 . org-ref-open-citation-at-point)
(16777327 . org-ref-cite-hydra/body) (follow-link . mouse-face)
(mouse-3 . org-find-file-at-mouse) (mouse-2 . org-open-at-mouse))
)
("citetitle" :follow
(lambda (_) (funcall org-ref-cite-onclick-function nil)) :export
org-ref-format-citetitle :complete org-citetitle-complete-link
:help-echo
(lambda (window object position)
(when org-ref-show-citation-on-enter
(save-excursion (goto-char position)
(let
((s (org-ref-format-entry (org-ref-get-bibtex-key-under-cursor)))
)
(with-temp-buffer (insert s) (fill-paragraph) (buffer-string)))
)
)
)
:face org-ref-cite-link-face-fn :display full :keymap
(keymap
(tab lambda nil (interactive)
(funcall org-ref-insert-cite-function))
(S-up . org-ref-sort-citation-link)
(S-right lambda nil (interactive) (org-ref-swap-citation-link 1))
(S-left lambda nil (interactive) (org-ref-swap-citation-link -1))
(C-right . org-ref-next-key) (C-left . org-ref-previous-key)
(16777337 lambda nil
"Paste key at point. Assumes the first thing in the killring is a key."
(interactive) (org-ref-insert-key-at-point (car kill-ring)))
(16777303 lambda nil "Copy all the keys at point." (interactive)
(kill-new (org-element-property :path (org-element-context))))
(16777335 lambda nil (interactive)
(kill-new (car (org-ref-get-bibtex-key-and-file))))
(16777318 lambda nil (interactive)
(save-excursion (org-ref-open-citation-at-point)
(kill-new (org-ref-format-bibtex-entry-at-point)))
)
(16777319 . org-ref-google-scholar-at-point)
(16777317 lambda nil "Email entry at point" (interactive)
(org-ref-open-citation-at-point) (org-ref-email-bibtex-entry))
(16777315 . org-ref-wos-citing-at-point)
(16777330 . org-ref-wos-related-at-point)
(16777326 . org-ref-open-notes-at-point)
(16777328 . org-ref-open-pdf-at-point)
(16777333 . org-ref-open-url-at-point)
(16777314 . org-ref-open-citation-at-point)
(16777327 . org-ref-cite-hydra/body) (follow-link . mouse-face)
(mouse-3 . org-find-file-at-mouse) (mouse-2 . org-open-at-mouse))
)
("Citeauthor*" :follow
(lambda (_) (funcall org-ref-cite-onclick-function nil)) :export
org-ref-format-Citeauthor* :complete org-Citeauthor*-complete-link
:help-echo
(lambda (window object position)
(when org-ref-show-citation-on-enter
(save-excursion (goto-char position)
(let
((s (org-ref-format-entry (org-ref-get-bibtex-key-under-cursor)))
)
(with-temp-buffer (insert s) (fill-paragraph) (buffer-string)))
)
)
)
:face org-ref-cite-link-face-fn :display full :keymap
(keymap
(tab lambda nil (interactive)
(funcall org-ref-insert-cite-function))
(S-up . org-ref-sort-citation-link)
(S-right lambda nil (interactive) (org-ref-swap-citation-link 1))
(S-left lambda nil (interactive) (org-ref-swap-citation-link -1))
(C-right . org-ref-next-key) (C-left . org-ref-previous-key)
(16777337 lambda nil
"Paste key at point. Assumes the first thing in the killring is a key."
(interactive) (org-ref-insert-key-at-point (car kill-ring)))
(16777303 lambda nil "Copy all the keys at point." (interactive)
(kill-new (org-element-property :path (org-element-context))))
(16777335 lambda nil (interactive)
(kill-new (car (org-ref-get-bibtex-key-and-file))))
(16777318 lambda nil (interactive)
(save-excursion (org-ref-open-citation-at-point)
(kill-new (org-ref-format-bibtex-entry-at-point)))
)
(16777319 . org-ref-google-scholar-at-point)
(16777317 lambda nil "Email entry at point" (interactive)
(org-ref-open-citation-at-point) (org-ref-email-bibtex-entry))
(16777315 . org-ref-wos-citing-at-point)
(16777330 . org-ref-wos-related-at-point)
(16777326 . org-ref-open-notes-at-point)
(16777328 . org-ref-open-pdf-at-point)
(16777333 . org-ref-open-url-at-point)
(16777314 . org-ref-open-citation-at-point)
(16777327 . org-ref-cite-hydra/body) (follow-link . mouse-face)
(mouse-3 . org-find-file-at-mouse) (mouse-2 . org-open-at-mouse))
)
("Autocite*" :follow
(lambda (_) (funcall org-ref-cite-onclick-function nil)) :export
org-ref-format-Autocite* :complete org-Autocite*-complete-link
:help-echo
(lambda (window object position)
(when org-ref-show-citation-on-enter
(save-excursion (goto-char position)
(let
((s (org-ref-format-entry (org-ref-get-bibtex-key-under-cursor)))
)
(with-temp-buffer (insert s) (fill-paragraph) (buffer-string)))
)
)
)
:face org-ref-cite-link-face-fn :display full :keymap
(keymap
(tab lambda nil (interactive)
(funcall org-ref-insert-cite-function))
(S-up . org-ref-sort-citation-link)
(S-right lambda nil (interactive) (org-ref-swap-citation-link 1))
(S-left lambda nil (interactive) (org-ref-swap-citation-link -1))
(C-right . org-ref-next-key) (C-left . org-ref-previous-key)
(16777337 lambda nil
"Paste key at point. Assumes the first thing in the killring is a key."
(interactive) (org-ref-insert-key-at-point (car kill-ring)))
(16777303 lambda nil "Copy all the keys at point." (interactive)
(kill-new (org-element-property :path (org-element-context))))
(16777335 lambda nil (interactive)
(kill-new (car (org-ref-get-bibtex-key-and-file))))
(16777318 lambda nil (interactive)
(save-excursion (org-ref-open-citation-at-point)
(kill-new (org-ref-format-bibtex-entry-at-point)))
)
(16777319 . org-ref-google-scholar-at-point)
(16777317 lambda nil "Email entry at point" (interactive)
(org-ref-open-citation-at-point) (org-ref-email-bibtex-entry))
(16777315 . org-ref-wos-citing-at-point)
(16777330 . org-ref-wos-related-at-point)
(16777326 . org-ref-open-notes-at-point)
(16777328 . org-ref-open-pdf-at-point)
(16777333 . org-ref-open-url-at-point)
(16777314 . org-ref-open-citation-at-point)
(16777327 . org-ref-cite-hydra/body) (follow-link . mouse-face)
(mouse-3 . org-find-file-at-mouse) (mouse-2 . org-open-at-mouse))
)
("autocite*" :follow
(lambda (_) (funcall org-ref-cite-onclick-function nil)) :export
org-ref-format-autocite* :complete org-autocite*-complete-link
:help-echo
(lambda (window object position)
(when org-ref-show-citation-on-enter
(save-excursion (goto-char position)
(let
((s (org-ref-format-entry (org-ref-get-bibtex-key-under-cursor)))
)
(with-temp-buffer (insert s) (fill-paragraph) (buffer-string)))
)
)
)
:face org-ref-cite-link-face-fn :display full :keymap
(keymap
(tab lambda nil (interactive)
(funcall org-ref-insert-cite-function))
(S-up . org-ref-sort-citation-link)
(S-right lambda nil (interactive) (org-ref-swap-citation-link 1))
(S-left lambda nil (interactive) (org-ref-swap-citation-link -1))
(C-right . org-ref-next-key) (C-left . org-ref-previous-key)
(16777337 lambda nil
"Paste key at point. Assumes the first thing in the killring is a key."
(interactive) (org-ref-insert-key-at-point (car kill-ring)))
(16777303 lambda nil "Copy all the keys at point." (interactive)
(kill-new (org-element-property :path (org-element-context))))
(16777335 lambda nil (interactive)
(kill-new (car (org-ref-get-bibtex-key-and-file))))
(16777318 lambda nil (interactive)
(save-excursion (org-ref-open-citation-at-point)
(kill-new (org-ref-format-bibtex-entry-at-point)))
)
(16777319 . org-ref-google-scholar-at-point)
(16777317 lambda nil "Email entry at point" (interactive)
(org-ref-open-citation-at-point) (org-ref-email-bibtex-entry))
(16777315 . org-ref-wos-citing-at-point)
(16777330 . org-ref-wos-related-at-point)
(16777326 . org-ref-open-notes-at-point)
(16777328 . org-ref-open-pdf-at-point)
(16777333 . org-ref-open-url-at-point)
(16777314 . org-ref-open-citation-at-point)
(16777327 . org-ref-cite-hydra/body) (follow-link . mouse-face)
(mouse-3 . org-find-file-at-mouse) (mouse-2 . org-open-at-mouse))
)
("Autocite" :follow
(lambda (_) (funcall org-ref-cite-onclick-function nil)) :export
org-ref-format-Autocite :complete org-Autocite-complete-link
:help-echo
(lambda (window object position)
(when org-ref-show-citation-on-enter
(save-excursion (goto-char position)
(let
((s (org-ref-format-entry (org-ref-get-bibtex-key-under-cursor)))
)
(with-temp-buffer (insert s) (fill-paragraph) (buffer-string)))
)
)
)
:face org-ref-cite-link-face-fn :display full :keymap
(keymap
(tab lambda nil (interactive)
(funcall org-ref-insert-cite-function))
(S-up . org-ref-sort-citation-link)
(S-right lambda nil (interactive) (org-ref-swap-citation-link 1))
(S-left lambda nil (interactive) (org-ref-swap-citation-link -1))
(C-right . org-ref-next-key) (C-left . org-ref-previous-key)
(16777337 lambda nil
"Paste key at point. Assumes the first thing in the killring is a key."
(interactive) (org-ref-insert-key-at-point (car kill-ring)))
(16777303 lambda nil "Copy all the keys at point." (interactive)
(kill-new (org-element-property :path (org-element-context))))
(16777335 lambda nil (interactive)
(kill-new (car (org-ref-get-bibtex-key-and-file))))
(16777318 lambda nil (interactive)
(save-excursion (org-ref-open-citation-at-point)
(kill-new (org-ref-format-bibtex-entry-at-point)))
)
(16777319 . org-ref-google-scholar-at-point)
(16777317 lambda nil "Email entry at point" (interactive)
(org-ref-open-citation-at-point) (org-ref-email-bibtex-entry))
(16777315 . org-ref-wos-citing-at-point)
(16777330 . org-ref-wos-related-at-point)
(16777326 . org-ref-open-notes-at-point)
(16777328 . org-ref-open-pdf-at-point)
(16777333 . org-ref-open-url-at-point)
(16777314 . org-ref-open-citation-at-point)
(16777327 . org-ref-cite-hydra/body) (follow-link . mouse-face)
(mouse-3 . org-find-file-at-mouse) (mouse-2 . org-open-at-mouse))
)
("autocite" :follow
(lambda (_) (funcall org-ref-cite-onclick-function nil)) :export
org-ref-format-autocite :complete org-autocite-complete-link
:help-echo
(lambda (window object position)
(when org-ref-show-citation-on-enter
(save-excursion (goto-char position)
(let
((s (org-ref-format-entry (org-ref-get-bibtex-key-under-cursor)))
)
(with-temp-buffer (insert s) (fill-paragraph) (buffer-string)))
)
)
)
:face org-ref-cite-link-face-fn :display full :keymap
(keymap
(tab lambda nil (interactive)
(funcall org-ref-insert-cite-function))
(S-up . org-ref-sort-citation-link)
(S-right lambda nil (interactive) (org-ref-swap-citation-link 1))
(S-left lambda nil (interactive) (org-ref-swap-citation-link -1))
(C-right . org-ref-next-key) (C-left . org-ref-previous-key)
(16777337 lambda nil
"Paste key at point. Assumes the first thing in the killring is a key."
(interactive) (org-ref-insert-key-at-point (car kill-ring)))
(16777303 lambda nil "Copy all the keys at point." (interactive)
(kill-new (org-element-property :path (org-element-context))))
(16777335 lambda nil (interactive)
(kill-new (car (org-ref-get-bibtex-key-and-file))))
(16777318 lambda nil (interactive)
(save-excursion (org-ref-open-citation-at-point)
(kill-new (org-ref-format-bibtex-entry-at-point)))
)
(16777319 . org-ref-google-scholar-at-point)
(16777317 lambda nil "Email entry at point" (interactive)
(org-ref-open-citation-at-point) (org-ref-email-bibtex-entry))
(16777315 . org-ref-wos-citing-at-point)
(16777330 . org-ref-wos-related-at-point)
(16777326 . org-ref-open-notes-at-point)
(16777328 . org-ref-open-pdf-at-point)
(16777333 . org-ref-open-url-at-point)
(16777314 . org-ref-open-citation-at-point)
(16777327 . org-ref-cite-hydra/body) (follow-link . mouse-face)
(mouse-3 . org-find-file-at-mouse) (mouse-2 . org-open-at-mouse))
)
("supercite" :follow
(lambda (_) (funcall org-ref-cite-onclick-function nil)) :export
org-ref-format-supercite :complete org-supercite-complete-link
:help-echo
(lambda (window object position)
(when org-ref-show-citation-on-enter
(save-excursion (goto-char position)
(let
((s (org-ref-format-entry (org-ref-get-bibtex-key-under-cursor)))
)
(with-temp-buffer (insert s) (fill-paragraph) (buffer-string)))
)
)
)
:face org-ref-cite-link-face-fn :display full :keymap
(keymap
(tab lambda nil (interactive)
(funcall org-ref-insert-cite-function))
(S-up . org-ref-sort-citation-link)
(S-right lambda nil (interactive) (org-ref-swap-citation-link 1))
(S-left lambda nil (interactive) (org-ref-swap-citation-link -1))
(C-right . org-ref-next-key) (C-left . org-ref-previous-key)
(16777337 lambda nil
"Paste key at point. Assumes the first thing in the killring is a key."
(interactive) (org-ref-insert-key-at-point (car kill-ring)))
(16777303 lambda nil "Copy all the keys at point." (interactive)
(kill-new (org-element-property :path (org-element-context))))
(16777335 lambda nil (interactive)
(kill-new (car (org-ref-get-bibtex-key-and-file))))
(16777318 lambda nil (interactive)
(save-excursion (org-ref-open-citation-at-point)
(kill-new (org-ref-format-bibtex-entry-at-point)))
)
(16777319 . org-ref-google-scholar-at-point)
(16777317 lambda nil "Email entry at point" (interactive)
(org-ref-open-citation-at-point) (org-ref-email-bibtex-entry))
(16777315 . org-ref-wos-citing-at-point)
(16777330 . org-ref-wos-related-at-point)
(16777326 . org-ref-open-notes-at-point)
(16777328 . org-ref-open-pdf-at-point)
(16777333 . org-ref-open-url-at-point)
(16777314 . org-ref-open-citation-at-point)
(16777327 . org-ref-cite-hydra/body) (follow-link . mouse-face)
(mouse-3 . org-find-file-at-mouse) (mouse-2 . org-open-at-mouse))
)
("parencite*" :follow
(lambda (_) (funcall org-ref-cite-onclick-function nil)) :export
org-ref-format-parencite* :complete org-parencite*-complete-link
:help-echo
(lambda (window object position)
(when org-ref-show-citation-on-enter
(save-excursion (goto-char position)
(let
((s (org-ref-format-entry (org-ref-get-bibtex-key-under-cursor)))
)
(with-temp-buffer (insert s) (fill-paragraph) (buffer-string)))
)
)
)
:face org-ref-cite-link-face-fn :display full :keymap
(keymap
(tab lambda nil (interactive)
(funcall org-ref-insert-cite-function))
(S-up . org-ref-sort-citation-link)
(S-right lambda nil (interactive) (org-ref-swap-citation-link 1))
(S-left lambda nil (interactive) (org-ref-swap-citation-link -1))
(C-right . org-ref-next-key) (C-left . org-ref-previous-key)
(16777337 lambda nil
"Paste key at point. Assumes the first thing in the killring is a key."
(interactive) (org-ref-insert-key-at-point (car kill-ring)))
(16777303 lambda nil "Copy all the keys at point." (interactive)
(kill-new (org-element-property :path (org-element-context))))
(16777335 lambda nil (interactive)
(kill-new (car (org-ref-get-bibtex-key-and-file))))
(16777318 lambda nil (interactive)
(save-excursion (org-ref-open-citation-at-point)
(kill-new (org-ref-format-bibtex-entry-at-point)))
)
(16777319 . org-ref-google-scholar-at-point)
(16777317 lambda nil "Email entry at point" (interactive)
(org-ref-open-citation-at-point) (org-ref-email-bibtex-entry))
(16777315 . org-ref-wos-citing-at-point)
(16777330 . org-ref-wos-related-at-point)
(16777326 . org-ref-open-notes-at-point)
(16777328 . org-ref-open-pdf-at-point)
(16777333 . org-ref-open-url-at-point)
(16777314 . org-ref-open-citation-at-point)
(16777327 . org-ref-cite-hydra/body) (follow-link . mouse-face)
(mouse-3 . org-find-file-at-mouse) (mouse-2 . org-open-at-mouse))
)
("cite*" :follow
(lambda (_) (funcall org-ref-cite-onclick-function nil)) :export
org-ref-format-cite* :complete org-cite*-complete-link :help-echo
(lambda (window object position)
(when org-ref-show-citation-on-enter
(save-excursion (goto-char position)
(let
((s (org-ref-format-entry (org-ref-get-bibtex-key-under-cursor)))
)
(with-temp-buffer (insert s) (fill-paragraph) (buffer-string)))
)
)
)
:face org-ref-cite-link-face-fn :display full :keymap
(keymap
(tab lambda nil (interactive)
(funcall org-ref-insert-cite-function))
(S-up . org-ref-sort-citation-link)
(S-right lambda nil (interactive) (org-ref-swap-citation-link 1))
(S-left lambda nil (interactive) (org-ref-swap-citation-link -1))
(C-right . org-ref-next-key) (C-left . org-ref-previous-key)
(16777337 lambda nil
"Paste key at point. Assumes the first thing in the killring is a key."
(interactive) (org-ref-insert-key-at-point (car kill-ring)))
(16777303 lambda nil "Copy all the keys at point." (interactive)
(kill-new (org-element-property :path (org-element-context))))
(16777335 lambda nil (interactive)
(kill-new (car (org-ref-get-bibtex-key-and-file))))
(16777318 lambda nil (interactive)
(save-excursion (org-ref-open-citation-at-point)
(kill-new (org-ref-format-bibtex-entry-at-point)))
)
(16777319 . org-ref-google-scholar-at-point)
(16777317 lambda nil "Email entry at point" (interactive)
(org-ref-open-citation-at-point) (org-ref-email-bibtex-entry))
(16777315 . org-ref-wos-citing-at-point)
(16777330 . org-ref-wos-related-at-point)
(16777326 . org-ref-open-notes-at-point)
(16777328 . org-ref-open-pdf-at-point)
(16777333 . org-ref-open-url-at-point)
(16777314 . org-ref-open-citation-at-point)
(16777327 . org-ref-cite-hydra/body) (follow-link . mouse-face)
(mouse-3 . org-find-file-at-mouse) (mouse-2 . org-open-at-mouse))
)
("Smartcite" :follow
(lambda (_) (funcall org-ref-cite-onclick-function nil)) :export
org-ref-format-Smartcite :complete org-Smartcite-complete-link
:help-echo
(lambda (window object position)
(when org-ref-show-citation-on-enter
(save-excursion (goto-char position)
(let
((s (org-ref-format-entry (org-ref-get-bibtex-key-under-cursor)))
)
(with-temp-buffer (insert s) (fill-paragraph) (buffer-string)))
)
)
)
:face org-ref-cite-link-face-fn :display full :keymap
(keymap
(tab lambda nil (interactive)
(funcall org-ref-insert-cite-function))
(S-up . org-ref-sort-citation-link)
(S-right lambda nil (interactive) (org-ref-swap-citation-link 1))
(S-left lambda nil (interactive) (org-ref-swap-citation-link -1))
(C-right . org-ref-next-key) (C-left . org-ref-previous-key)
(16777337 lambda nil
"Paste key at point. Assumes the first thing in the killring is a key."
(interactive) (org-ref-insert-key-at-point (car kill-ring)))
(16777303 lambda nil "Copy all the keys at point." (interactive)
(kill-new (org-element-property :path (org-element-context))))
(16777335 lambda nil (interactive)
(kill-new (car (org-ref-get-bibtex-key-and-file))))
(16777318 lambda nil (interactive)
(save-excursion (org-ref-open-citation-at-point)
(kill-new (org-ref-format-bibtex-entry-at-point)))
)
(16777319 . org-ref-google-scholar-at-point)
(16777317 lambda nil "Email entry at point" (interactive)
(org-ref-open-citation-at-point) (org-ref-email-bibtex-entry))
(16777315 . org-ref-wos-citing-at-point)
(16777330 . org-ref-wos-related-at-point)
(16777326 . org-ref-open-notes-at-point)
(16777328 . org-ref-open-pdf-at-point)
(16777333 . org-ref-open-url-at-point)
(16777314 . org-ref-open-citation-at-point)
(16777327 . org-ref-cite-hydra/body) (follow-link . mouse-face)
(mouse-3 . org-find-file-at-mouse) (mouse-2 . org-open-at-mouse))
)
("smartcite" :follow
(lambda (_) (funcall org-ref-cite-onclick-function nil)) :export
org-ref-format-smartcite :complete org-smartcite-complete-link
:help-echo
(lambda (window object position)
(when org-ref-show-citation-on-enter
(save-excursion (goto-char position)
(let
((s (org-ref-format-entry (org-ref-get-bibtex-key-under-cursor)))
)
(with-temp-buffer (insert s) (fill-paragraph) (buffer-string)))
)
)
)
:face org-ref-cite-link-face-fn :display full :keymap
(keymap
(tab lambda nil (interactive)
(funcall org-ref-insert-cite-function))
(S-up . org-ref-sort-citation-link)
(S-right lambda nil (interactive) (org-ref-swap-citation-link 1))
(S-left lambda nil (interactive) (org-ref-swap-citation-link -1))
(C-right . org-ref-next-key) (C-left . org-ref-previous-key)
(16777337 lambda nil
"Paste key at point. Assumes the first thing in the killring is a key."
(interactive) (org-ref-insert-key-at-point (car kill-ring)))
(16777303 lambda nil "Copy all the keys at point." (interactive)
(kill-new (org-element-property :path (org-element-context))))
(16777335 lambda nil (interactive)
(kill-new (car (org-ref-get-bibtex-key-and-file))))
(16777318 lambda nil (interactive)
(save-excursion (org-ref-open-citation-at-point)
(kill-new (org-ref-format-bibtex-entry-at-point)))
)
(16777319 . org-ref-google-scholar-at-point)
(16777317 lambda nil "Email entry at point" (interactive)
(org-ref-open-citation-at-point) (org-ref-email-bibtex-entry))
(16777315 . org-ref-wos-citing-at-point)
(16777330 . org-ref-wos-related-at-point)
(16777326 . org-ref-open-notes-at-point)
(16777328 . org-ref-open-pdf-at-point)
(16777333 . org-ref-open-url-at-point)
(16777314 . org-ref-open-citation-at-point)
(16777327 . org-ref-cite-hydra/body) (follow-link . mouse-face)
(mouse-3 . org-find-file-at-mouse) (mouse-2 . org-open-at-mouse))
)
("Textcite" :follow
(lambda (_) (funcall org-ref-cite-onclick-function nil)) :export
org-ref-format-Textcite :complete org-Textcite-complete-link
:help-echo
(lambda (window object position)
(when org-ref-show-citation-on-enter
(save-excursion (goto-char position)
(let
((s (org-ref-format-entry (org-ref-get-bibtex-key-under-cursor)))
)
(with-temp-buffer (insert s) (fill-paragraph) (buffer-string)))
)
)
)
:face org-ref-cite-link-face-fn :display full :keymap
(keymap
(tab lambda nil (interactive)
(funcall org-ref-insert-cite-function))
(S-up . org-ref-sort-citation-link)
(S-right lambda nil (interactive) (org-ref-swap-citation-link 1))
(S-left lambda nil (interactive) (org-ref-swap-citation-link -1))
(C-right . org-ref-next-key) (C-left . org-ref-previous-key)
(16777337 lambda nil
"Paste key at point. Assumes the first thing in the killring is a key."
(interactive) (org-ref-insert-key-at-point (car kill-ring)))
(16777303 lambda nil "Copy all the keys at point." (interactive)
(kill-new (org-element-property :path (org-element-context))))
(16777335 lambda nil (interactive)
(kill-new (car (org-ref-get-bibtex-key-and-file))))
(16777318 lambda nil (interactive)
(save-excursion (org-ref-open-citation-at-point)
(kill-new (org-ref-format-bibtex-entry-at-point)))
)
(16777319 . org-ref-google-scholar-at-point)
(16777317 lambda nil "Email entry at point" (interactive)
(org-ref-open-citation-at-point) (org-ref-email-bibtex-entry))
(16777315 . org-ref-wos-citing-at-point)
(16777330 . org-ref-wos-related-at-point)
(16777326 . org-ref-open-notes-at-point)
(16777328 . org-ref-open-pdf-at-point)
(16777333 . org-ref-open-url-at-point)
(16777314 . org-ref-open-citation-at-point)
(16777327 . org-ref-cite-hydra/body) (follow-link . mouse-face)
(mouse-3 . org-find-file-at-mouse) (mouse-2 . org-open-at-mouse))
)
("textcite" :follow
(lambda (_) (funcall org-ref-cite-onclick-function nil)) :export
org-ref-format-textcite :complete org-textcite-complete-link
:help-echo
(lambda (window object position)
(when org-ref-show-citation-on-enter
(save-excursion (goto-char position)
(let
((s (org-ref-format-entry (org-ref-get-bibtex-key-under-cursor)))
)
(with-temp-buffer (insert s) (fill-paragraph) (buffer-string)))
)
)
)
:face org-ref-cite-link-face-fn :display full :keymap
(keymap
(tab lambda nil (interactive)
(funcall org-ref-insert-cite-function))
(S-up . org-ref-sort-citation-link)
(S-right lambda nil (interactive) (org-ref-swap-citation-link 1))
(S-left lambda nil (interactive) (org-ref-swap-citation-link -1))
(C-right . org-ref-next-key) (C-left . org-ref-previous-key)
(16777337 lambda nil
"Paste key at point. Assumes the first thing in the killring is a key."
(interactive) (org-ref-insert-key-at-point (car kill-ring)))
(16777303 lambda nil "Copy all the keys at point." (interactive)
(kill-new (org-element-property :path (org-element-context))))
(16777335 lambda nil (interactive)
(kill-new (car (org-ref-get-bibtex-key-and-file))))
(16777318 lambda nil (interactive)
(save-excursion (org-ref-open-citation-at-point)
(kill-new (org-ref-format-bibtex-entry-at-point)))
)
(16777319 . org-ref-google-scholar-at-point)
(16777317 lambda nil "Email entry at point" (interactive)
(org-ref-open-citation-at-point) (org-ref-email-bibtex-entry))
(16777315 . org-ref-wos-citing-at-point)
(16777330 . org-ref-wos-related-at-point)
(16777326 . org-ref-open-notes-at-point)
(16777328 . org-ref-open-pdf-at-point)
(16777333 . org-ref-open-url-at-point)
(16777314 . org-ref-open-citation-at-point)
(16777327 . org-ref-cite-hydra/body) (follow-link . mouse-face)
(mouse-3 . org-find-file-at-mouse) (mouse-2 . org-open-at-mouse))
)
("footcitetext" :follow
(lambda (_) (funcall org-ref-cite-onclick-function nil)) :export
org-ref-format-footcitetext :complete org-footcitetext-complete-link
:help-echo
(lambda (window object position)
(when org-ref-show-citation-on-enter
(save-excursion (goto-char position)
(let
((s (org-ref-format-entry (org-ref-get-bibtex-key-under-cursor)))
)
(with-temp-buffer (insert s) (fill-paragraph) (buffer-string)))
)
)
)
:face org-ref-cite-link-face-fn :display full :keymap
(keymap
(tab lambda nil (interactive)
(funcall org-ref-insert-cite-function))
(S-up . org-ref-sort-citation-link)
(S-right lambda nil (interactive) (org-ref-swap-citation-link 1))
(S-left lambda nil (interactive) (org-ref-swap-citation-link -1))
(C-right . org-ref-next-key) (C-left . org-ref-previous-key)
(16777337 lambda nil
"Paste key at point. Assumes the first thing in the killring is a key."
(interactive) (org-ref-insert-key-at-point (car kill-ring)))
(16777303 lambda nil "Copy all the keys at point." (interactive)
(kill-new (org-element-property :path (org-element-context))))
(16777335 lambda nil (interactive)
(kill-new (car (org-ref-get-bibtex-key-and-file))))
(16777318 lambda nil (interactive)
(save-excursion (org-ref-open-citation-at-point)
(kill-new (org-ref-format-bibtex-entry-at-point)))
)
(16777319 . org-ref-google-scholar-at-point)
(16777317 lambda nil "Email entry at point" (interactive)
(org-ref-open-citation-at-point) (org-ref-email-bibtex-entry))
(16777315 . org-ref-wos-citing-at-point)
(16777330 . org-ref-wos-related-at-point)
(16777326 . org-ref-open-notes-at-point)
(16777328 . org-ref-open-pdf-at-point)
(16777333 . org-ref-open-url-at-point)
(16777314 . org-ref-open-citation-at-point)
(16777327 . org-ref-cite-hydra/body) (follow-link . mouse-face)
(mouse-3 . org-find-file-at-mouse) (mouse-2 . org-open-at-mouse))
)
("footcite" :follow
(lambda (_) (funcall org-ref-cite-onclick-function nil)) :export
org-ref-format-footcite :complete org-footcite-complete-link
:help-echo
(lambda (window object position)
(when org-ref-show-citation-on-enter
(save-excursion (goto-char position)
(let
((s (org-ref-format-entry (org-ref-get-bibtex-key-under-cursor)))
)
(with-temp-buffer (insert s) (fill-paragraph) (buffer-string)))
)
)
)
:face org-ref-cite-link-face-fn :display full :keymap
(keymap
(tab lambda nil (interactive)
(funcall org-ref-insert-cite-function))
(S-up . org-ref-sort-citation-link)
(S-right lambda nil (interactive) (org-ref-swap-citation-link 1))
(S-left lambda nil (interactive) (org-ref-swap-citation-link -1))
(C-right . org-ref-next-key) (C-left . org-ref-previous-key)
(16777337 lambda nil
"Paste key at point. Assumes the first thing in the killring is a key."
(interactive) (org-ref-insert-key-at-point (car kill-ring)))
(16777303 lambda nil "Copy all the keys at point." (interactive)
(kill-new (org-element-property :path (org-element-context))))
(16777335 lambda nil (interactive)
(kill-new (car (org-ref-get-bibtex-key-and-file))))
(16777318 lambda nil (interactive)
(save-excursion (org-ref-open-citation-at-point)
(kill-new (org-ref-format-bibtex-entry-at-point)))
)
(16777319 . org-ref-google-scholar-at-point)
(16777317 lambda nil "Email entry at point" (interactive)
(org-ref-open-citation-at-point) (org-ref-email-bibtex-entry))
(16777315 . org-ref-wos-citing-at-point)
(16777330 . org-ref-wos-related-at-point)
(16777326 . org-ref-open-notes-at-point)
(16777328 . org-ref-open-pdf-at-point)
(16777333 . org-ref-open-url-at-point)
(16777314 . org-ref-open-citation-at-point)
(16777327 . org-ref-cite-hydra/body) (follow-link . mouse-face)
(mouse-3 . org-find-file-at-mouse) (mouse-2 . org-open-at-mouse))
)
("Parencite" :follow
(lambda (_) (funcall org-ref-cite-onclick-function nil)) :export
org-ref-format-Parencite :complete org-Parencite-complete-link
:help-echo
(lambda (window object position)
(when org-ref-show-citation-on-enter
(save-excursion (goto-char position)
(let
((s (org-ref-format-entry (org-ref-get-bibtex-key-under-cursor)))
)
(with-temp-buffer (insert s) (fill-paragraph) (buffer-string)))
)
)
)
:face org-ref-cite-link-face-fn :display full :keymap
(keymap
(tab lambda nil (interactive)
(funcall org-ref-insert-cite-function))
(S-up . org-ref-sort-citation-link)
(S-right lambda nil (interactive) (org-ref-swap-citation-link 1))
(S-left lambda nil (interactive) (org-ref-swap-citation-link -1))
(C-right . org-ref-next-key) (C-left . org-ref-previous-key)
(16777337 lambda nil
"Paste key at point. Assumes the first thing in the killring is a key."
(interactive) (org-ref-insert-key-at-point (car kill-ring)))
(16777303 lambda nil "Copy all the keys at point." (interactive)
(kill-new (org-element-property :path (org-element-context))))
(16777335 lambda nil (interactive)
(kill-new (car (org-ref-get-bibtex-key-and-file))))
(16777318 lambda nil (interactive)
(save-excursion (org-ref-open-citation-at-point)
(kill-new (org-ref-format-bibtex-entry-at-point)))
)
(16777319 . org-ref-google-scholar-at-point)
(16777317 lambda nil "Email entry at point" (interactive)
(org-ref-open-citation-at-point) (org-ref-email-bibtex-entry))
(16777315 . org-ref-wos-citing-at-point)
(16777330 . org-ref-wos-related-at-point)
(16777326 . org-ref-open-notes-at-point)
(16777328 . org-ref-open-pdf-at-point)
(16777333 . org-ref-open-url-at-point)
(16777314 . org-ref-open-citation-at-point)
(16777327 . org-ref-cite-hydra/body) (follow-link . mouse-face)
(mouse-3 . org-find-file-at-mouse) (mouse-2 . org-open-at-mouse))
)
("parencite" :follow
(lambda (_) (funcall org-ref-cite-onclick-function nil)) :export
org-ref-format-parencite :complete org-parencite-complete-link
:help-echo
(lambda (window object position)
(when org-ref-show-citation-on-enter
(save-excursion (goto-char position)
(let
((s (org-ref-format-entry (org-ref-get-bibtex-key-under-cursor)))
)
(with-temp-buffer (insert s) (fill-paragraph) (buffer-string)))
)
)
)
:face org-ref-cite-link-face-fn :display full :keymap
(keymap
(tab lambda nil (interactive)
(funcall org-ref-insert-cite-function))
(S-up . org-ref-sort-citation-link)
(S-right lambda nil (interactive) (org-ref-swap-citation-link 1))
(S-left lambda nil (interactive) (org-ref-swap-citation-link -1))
(C-right . org-ref-next-key) (C-left . org-ref-previous-key)
(16777337 lambda nil
"Paste key at point. Assumes the first thing in the killring is a key."
(interactive) (org-ref-insert-key-at-point (car kill-ring)))
(16777303 lambda nil "Copy all the keys at point." (interactive)
(kill-new (org-element-property :path (org-element-context))))
(16777335 lambda nil (interactive)
(kill-new (car (org-ref-get-bibtex-key-and-file))))
(16777318 lambda nil (interactive)
(save-excursion (org-ref-open-citation-at-point)
(kill-new (org-ref-format-bibtex-entry-at-point)))
)
(16777319 . org-ref-google-scholar-at-point)
(16777317 lambda nil "Email entry at point" (interactive)
(org-ref-open-citation-at-point) (org-ref-email-bibtex-entry))
(16777315 . org-ref-wos-citing-at-point)
(16777330 . org-ref-wos-related-at-point)
(16777326 . org-ref-open-notes-at-point)
(16777328 . org-ref-open-pdf-at-point)
(16777333 . org-ref-open-url-at-point)
(16777314 . org-ref-open-citation-at-point)
(16777327 . org-ref-cite-hydra/body) (follow-link . mouse-face)
(mouse-3 . org-find-file-at-mouse) (mouse-2 . org-open-at-mouse))
)
("Cite" :follow
(lambda (_) (funcall org-ref-cite-onclick-function nil)) :export
org-ref-format-Cite :complete org-Cite-complete-link :help-echo
(lambda (window object position)
(when org-ref-show-citation-on-enter
(save-excursion (goto-char position)
(let
((s (org-ref-format-entry (org-ref-get-bibtex-key-under-cursor)))
)
(with-temp-buffer (insert s) (fill-paragraph) (buffer-string)))
)
)
)
:face org-ref-cite-link-face-fn :display full :keymap
(keymap
(tab lambda nil (interactive)
(funcall org-ref-insert-cite-function))
(S-up . org-ref-sort-citation-link)
(S-right lambda nil (interactive) (org-ref-swap-citation-link 1))
(S-left lambda nil (interactive) (org-ref-swap-citation-link -1))
(C-right . org-ref-next-key) (C-left . org-ref-previous-key)
(16777337 lambda nil
"Paste key at point. Assumes the first thing in the killring is a key."
(interactive) (org-ref-insert-key-at-point (car kill-ring)))
(16777303 lambda nil "Copy all the keys at point." (interactive)
(kill-new (org-element-property :path (org-element-context))))
(16777335 lambda nil (interactive)
(kill-new (car (org-ref-get-bibtex-key-and-file))))
(16777318 lambda nil (interactive)
(save-excursion (org-ref-open-citation-at-point)
(kill-new (org-ref-format-bibtex-entry-at-point)))
)
(16777319 . org-ref-google-scholar-at-point)
(16777317 lambda nil "Email entry at point" (interactive)
(org-ref-open-citation-at-point) (org-ref-email-bibtex-entry))
(16777315 . org-ref-wos-citing-at-point)
(16777330 . org-ref-wos-related-at-point)
(16777326 . org-ref-open-notes-at-point)
(16777328 . org-ref-open-pdf-at-point)
(16777333 . org-ref-open-url-at-point)
(16777314 . org-ref-open-citation-at-point)
(16777327 . org-ref-cite-hydra/body) (follow-link . mouse-face)
(mouse-3 . org-find-file-at-mouse) (mouse-2 . org-open-at-mouse))
)
("Citeauthor" :follow
(lambda (_) (funcall org-ref-cite-onclick-function nil)) :export
org-ref-format-Citeauthor :complete org-Citeauthor-complete-link
:help-echo
(lambda (window object position)
(when org-ref-show-citation-on-enter
(save-excursion (goto-char position)
(let
((s (org-ref-format-entry (org-ref-get-bibtex-key-under-cursor)))
)
(with-temp-buffer (insert s) (fill-paragraph) (buffer-string)))
)
)
)
:face org-ref-cite-link-face-fn :display full :keymap
(keymap
(tab lambda nil (interactive)
(funcall org-ref-insert-cite-function))
(S-up . org-ref-sort-citation-link)
(S-right lambda nil (interactive) (org-ref-swap-citation-link 1))
(S-left lambda nil (interactive) (org-ref-swap-citation-link -1))
(C-right . org-ref-next-key) (C-left . org-ref-previous-key)
(16777337 lambda nil
"Paste key at point. Assumes the first thing in the killring is a key."
(interactive) (org-ref-insert-key-at-point (car kill-ring)))
(16777303 lambda nil "Copy all the keys at point." (interactive)
(kill-new (org-element-property :path (org-element-context))))
(16777335 lambda nil (interactive)
(kill-new (car (org-ref-get-bibtex-key-and-file))))
(16777318 lambda nil (interactive)
(save-excursion (org-ref-open-citation-at-point)
(kill-new (org-ref-format-bibtex-entry-at-point)))
)
(16777319 . org-ref-google-scholar-at-point)
(16777317 lambda nil "Email entry at point" (interactive)
(org-ref-open-citation-at-point) (org-ref-email-bibtex-entry))
(16777315 . org-ref-wos-citing-at-point)
(16777330 . org-ref-wos-related-at-point)
(16777326 . org-ref-open-notes-at-point)
(16777328 . org-ref-open-pdf-at-point)
(16777333 . org-ref-open-url-at-point)
(16777314 . org-ref-open-citation-at-point)
(16777327 . org-ref-cite-hydra/body) (follow-link . mouse-face)
(mouse-3 . org-find-file-at-mouse) (mouse-2 . org-open-at-mouse))
)
("Citealp" :follow
(lambda (_) (funcall org-ref-cite-onclick-function nil)) :export
org-ref-format-Citealp :complete org-Citealp-complete-link
:help-echo
(lambda (window object position)
(when org-ref-show-citation-on-enter
(save-excursion (goto-char position)
(let
((s (org-ref-format-entry (org-ref-get-bibtex-key-under-cursor)))
)
(with-temp-buffer (insert s) (fill-paragraph) (buffer-string)))
)
)
)
:face org-ref-cite-link-face-fn :display full :keymap
(keymap
(tab lambda nil (interactive)
(funcall org-ref-insert-cite-function))
(S-up . org-ref-sort-citation-link)
(S-right lambda nil (interactive) (org-ref-swap-citation-link 1))
(S-left lambda nil (interactive) (org-ref-swap-citation-link -1))
(C-right . org-ref-next-key) (C-left . org-ref-previous-key)
(16777337 lambda nil
"Paste key at point. Assumes the first thing in the killring is a key."
(interactive) (org-ref-insert-key-at-point (car kill-ring)))
(16777303 lambda nil "Copy all the keys at point." (interactive)
(kill-new (org-element-property :path (org-element-context))))
(16777335 lambda nil (interactive)
(kill-new (car (org-ref-get-bibtex-key-and-file))))
(16777318 lambda nil (interactive)
(save-excursion (org-ref-open-citation-at-point)
(kill-new (org-ref-format-bibtex-entry-at-point)))
)
(16777319 . org-ref-google-scholar-at-point)
(16777317 lambda nil "Email entry at point" (interactive)
(org-ref-open-citation-at-point) (org-ref-email-bibtex-entry))
(16777315 . org-ref-wos-citing-at-point)
(16777330 . org-ref-wos-related-at-point)
(16777326 . org-ref-open-notes-at-point)
(16777328 . org-ref-open-pdf-at-point)
(16777333 . org-ref-open-url-at-point)
(16777314 . org-ref-open-citation-at-point)
(16777327 . org-ref-cite-hydra/body) (follow-link . mouse-face)
(mouse-3 . org-find-file-at-mouse) (mouse-2 . org-open-at-mouse))
)
("Citealt" :follow
(lambda (_) (funcall org-ref-cite-onclick-function nil)) :export
org-ref-format-Citealt :complete org-Citealt-complete-link
:help-echo
(lambda (window object position)
(when org-ref-show-citation-on-enter
(save-excursion (goto-char position)
(let
((s (org-ref-format-entry (org-ref-get-bibtex-key-under-cursor)))
)
(with-temp-buffer (insert s) (fill-paragraph) (buffer-string)))
)
)
)
:face org-ref-cite-link-face-fn :display full :keymap
(keymap
(tab lambda nil (interactive)
(funcall org-ref-insert-cite-function))
(S-up . org-ref-sort-citation-link)
(S-right lambda nil (interactive) (org-ref-swap-citation-link 1))
(S-left lambda nil (interactive) (org-ref-swap-citation-link -1))
(C-right . org-ref-next-key) (C-left . org-ref-previous-key)
(16777337 lambda nil
"Paste key at point. Assumes the first thing in the killring is a key."
(interactive) (org-ref-insert-key-at-point (car kill-ring)))
(16777303 lambda nil "Copy all the keys at point." (interactive)
(kill-new (org-element-property :path (org-element-context))))
(16777335 lambda nil (interactive)
(kill-new (car (org-ref-get-bibtex-key-and-file))))
(16777318 lambda nil (interactive)
(save-excursion (org-ref-open-citation-at-point)
(kill-new (org-ref-format-bibtex-entry-at-point)))
)
(16777319 . org-ref-google-scholar-at-point)
(16777317 lambda nil "Email entry at point" (interactive)
(org-ref-open-citation-at-point) (org-ref-email-bibtex-entry))
(16777315 . org-ref-wos-citing-at-point)
(16777330 . org-ref-wos-related-at-point)
(16777326 . org-ref-open-notes-at-point)
(16777328 . org-ref-open-pdf-at-point)
(16777333 . org-ref-open-url-at-point)
(16777314 . org-ref-open-citation-at-point)
(16777327 . org-ref-cite-hydra/body) (follow-link . mouse-face)
(mouse-3 . org-find-file-at-mouse) (mouse-2 . org-open-at-mouse))
)
("Citep" :follow
(lambda (_) (funcall org-ref-cite-onclick-function nil)) :export
org-ref-format-Citep :complete org-Citep-complete-link :help-echo
(lambda (window object position)
(when org-ref-show-citation-on-enter
(save-excursion (goto-char position)
(let
((s (org-ref-format-entry (org-ref-get-bibtex-key-under-cursor)))
)
(with-temp-buffer (insert s) (fill-paragraph) (buffer-string)))
)
)
)
:face org-ref-cite-link-face-fn :display full :keymap
(keymap
(tab lambda nil (interactive)
(funcall org-ref-insert-cite-function))
(S-up . org-ref-sort-citation-link)
(S-right lambda nil (interactive) (org-ref-swap-citation-link 1))
(S-left lambda nil (interactive) (org-ref-swap-citation-link -1))
(C-right . org-ref-next-key) (C-left . org-ref-previous-key)
(16777337 lambda nil
"Paste key at point. Assumes the first thing in the killring is a key."
(interactive) (org-ref-insert-key-at-point (car kill-ring)))
(16777303 lambda nil "Copy all the keys at point." (interactive)
(kill-new (org-element-property :path (org-element-context))))
(16777335 lambda nil (interactive)
(kill-new (car (org-ref-get-bibtex-key-and-file))))
(16777318 lambda nil (interactive)
(save-excursion (org-ref-open-citation-at-point)
(kill-new (org-ref-format-bibtex-entry-at-point)))
)
(16777319 . org-ref-google-scholar-at-point)
(16777317 lambda nil "Email entry at point" (interactive)
(org-ref-open-citation-at-point) (org-ref-email-bibtex-entry))
(16777315 . org-ref-wos-citing-at-point)
(16777330 . org-ref-wos-related-at-point)
(16777326 . org-ref-open-notes-at-point)
(16777328 . org-ref-open-pdf-at-point)
(16777333 . org-ref-open-url-at-point)
(16777314 . org-ref-open-citation-at-point)
(16777327 . org-ref-cite-hydra/body) (follow-link . mouse-face)
(mouse-3 . org-find-file-at-mouse) (mouse-2 . org-open-at-mouse))
)
("Citet" :follow
(lambda (_) (funcall org-ref-cite-onclick-function nil)) :export
org-ref-format-Citet :complete org-Citet-complete-link :help-echo
(lambda (window object position)
(when org-ref-show-citation-on-enter
(save-excursion (goto-char position)
(let
((s (org-ref-format-entry (org-ref-get-bibtex-key-under-cursor)))
)
(with-temp-buffer (insert s) (fill-paragraph) (buffer-string)))
)
)
)
:face org-ref-cite-link-face-fn :display full :keymap
(keymap
(tab lambda nil (interactive)
(funcall org-ref-insert-cite-function))
(S-up . org-ref-sort-citation-link)
(S-right lambda nil (interactive) (org-ref-swap-citation-link 1))
(S-left lambda nil (interactive) (org-ref-swap-citation-link -1))
(C-right . org-ref-next-key) (C-left . org-ref-previous-key)
(16777337 lambda nil
"Paste key at point. Assumes the first thing in the killring is a key."
(interactive) (org-ref-insert-key-at-point (car kill-ring)))
(16777303 lambda nil "Copy all the keys at point." (interactive)
(kill-new (org-element-property :path (org-element-context))))
(16777335 lambda nil (interactive)
(kill-new (car (org-ref-get-bibtex-key-and-file))))
(16777318 lambda nil (interactive)
(save-excursion (org-ref-open-citation-at-point)
(kill-new (org-ref-format-bibtex-entry-at-point)))
)
(16777319 . org-ref-google-scholar-at-point)
(16777317 lambda nil "Email entry at point" (interactive)
(org-ref-open-citation-at-point) (org-ref-email-bibtex-entry))
(16777315 . org-ref-wos-citing-at-point)
(16777330 . org-ref-wos-related-at-point)
(16777326 . org-ref-open-notes-at-point)
(16777328 . org-ref-open-pdf-at-point)
(16777333 . org-ref-open-url-at-point)
(16777314 . org-ref-open-citation-at-point)
(16777327 . org-ref-cite-hydra/body) (follow-link . mouse-face)
(mouse-3 . org-find-file-at-mouse) (mouse-2 . org-open-at-mouse))
)
("citeyearpar" :follow
(lambda (_) (funcall org-ref-cite-onclick-function nil)) :export
org-ref-format-citeyearpar :complete org-citeyearpar-complete-link
:help-echo
(lambda (window object position)
(when org-ref-show-citation-on-enter
(save-excursion (goto-char position)
(let
((s (org-ref-format-entry (org-ref-get-bibtex-key-under-cursor)))
)
(with-temp-buffer (insert s) (fill-paragraph) (buffer-string)))
)
)
)
:face org-ref-cite-link-face-fn :display full :keymap
(keymap
(tab lambda nil (interactive)
(funcall org-ref-insert-cite-function))
(S-up . org-ref-sort-citation-link)
(S-right lambda nil (interactive) (org-ref-swap-citation-link 1))
(S-left lambda nil (interactive) (org-ref-swap-citation-link -1))
(C-right . org-ref-next-key) (C-left . org-ref-previous-key)
(16777337 lambda nil
"Paste key at point. Assumes the first thing in the killring is a key."
(interactive) (org-ref-insert-key-at-point (car kill-ring)))
(16777303 lambda nil "Copy all the keys at point." (interactive)
(kill-new (org-element-property :path (org-element-context))))
(16777335 lambda nil (interactive)
(kill-new (car (org-ref-get-bibtex-key-and-file))))
(16777318 lambda nil (interactive)
(save-excursion (org-ref-open-citation-at-point)
(kill-new (org-ref-format-bibtex-entry-at-point)))
)
(16777319 . org-ref-google-scholar-at-point)
(16777317 lambda nil "Email entry at point" (interactive)
(org-ref-open-citation-at-point) (org-ref-email-bibtex-entry))
(16777315 . org-ref-wos-citing-at-point)
(16777330 . org-ref-wos-related-at-point)
(16777326 . org-ref-open-notes-at-point)
(16777328 . org-ref-open-pdf-at-point)
(16777333 . org-ref-open-url-at-point)
(16777314 . org-ref-open-citation-at-point)
(16777327 . org-ref-cite-hydra/body) (follow-link . mouse-face)
(mouse-3 . org-find-file-at-mouse) (mouse-2 . org-open-at-mouse))
)
("citeyear*" :follow
(lambda (_) (funcall org-ref-cite-onclick-function nil)) :export
org-ref-format-citeyear* :complete org-citeyear*-complete-link
:help-echo
(lambda (window object position)
(when org-ref-show-citation-on-enter
(save-excursion (goto-char position)
(let
((s (org-ref-format-entry (org-ref-get-bibtex-key-under-cursor)))
)
(with-temp-buffer (insert s) (fill-paragraph) (buffer-string)))
)
)
)
:face org-ref-cite-link-face-fn :display full :keymap
(keymap
(tab lambda nil (interactive)
(funcall org-ref-insert-cite-function))
(S-up . org-ref-sort-citation-link)
(S-right lambda nil (interactive) (org-ref-swap-citation-link 1))
(S-left lambda nil (interactive) (org-ref-swap-citation-link -1))
(C-right . org-ref-next-key) (C-left . org-ref-previous-key)
(16777337 lambda nil
"Paste key at point. Assumes the first thing in the killring is a key."
(interactive) (org-ref-insert-key-at-point (car kill-ring)))
(16777303 lambda nil "Copy all the keys at point." (interactive)
(kill-new (org-element-property :path (org-element-context))))
(16777335 lambda nil (interactive)
(kill-new (car (org-ref-get-bibtex-key-and-file))))
(16777318 lambda nil (interactive)
(save-excursion (org-ref-open-citation-at-point)
(kill-new (org-ref-format-bibtex-entry-at-point)))
)
(16777319 . org-ref-google-scholar-at-point)
(16777317 lambda nil "Email entry at point" (interactive)
(org-ref-open-citation-at-point) (org-ref-email-bibtex-entry))
(16777315 . org-ref-wos-citing-at-point)
(16777330 . org-ref-wos-related-at-point)
(16777326 . org-ref-open-notes-at-point)
(16777328 . org-ref-open-pdf-at-point)
(16777333 . org-ref-open-url-at-point)
(16777314 . org-ref-open-citation-at-point)
(16777327 . org-ref-cite-hydra/body) (follow-link . mouse-face)
(mouse-3 . org-find-file-at-mouse) (mouse-2 . org-open-at-mouse))
)
("citeyear" :follow
(lambda (_) (funcall org-ref-cite-onclick-function nil)) :export
org-ref-format-citeyear :complete org-citeyear-complete-link
:help-echo
(lambda (window object position)
(when org-ref-show-citation-on-enter
(save-excursion (goto-char position)
(let
((s (org-ref-format-entry (org-ref-get-bibtex-key-under-cursor)))
)
(with-temp-buffer (insert s) (fill-paragraph) (buffer-string)))
)
)
)
:face org-ref-cite-link-face-fn :display full :keymap
(keymap
(tab lambda nil (interactive)
(funcall org-ref-insert-cite-function))
(S-up . org-ref-sort-citation-link)
(S-right lambda nil (interactive) (org-ref-swap-citation-link 1))
(S-left lambda nil (interactive) (org-ref-swap-citation-link -1))
(C-right . org-ref-next-key) (C-left . org-ref-previous-key)
(16777337 lambda nil
"Paste key at point. Assumes the first thing in the killring is a key."
(interactive) (org-ref-insert-key-at-point (car kill-ring)))
(16777303 lambda nil "Copy all the keys at point." (interactive)
(kill-new (org-element-property :path (org-element-context))))
(16777335 lambda nil (interactive)
(kill-new (car (org-ref-get-bibtex-key-and-file))))
(16777318 lambda nil (interactive)
(save-excursion (org-ref-open-citation-at-point)
(kill-new (org-ref-format-bibtex-entry-at-point)))
)
(16777319 . org-ref-google-scholar-at-point)
(16777317 lambda nil "Email entry at point" (interactive)
(org-ref-open-citation-at-point) (org-ref-email-bibtex-entry))
(16777315 . org-ref-wos-citing-at-point)
(16777330 . org-ref-wos-related-at-point)
(16777326 . org-ref-open-notes-at-point)
(16777328 . org-ref-open-pdf-at-point)
(16777333 . org-ref-open-url-at-point)
(16777314 . org-ref-open-citation-at-point)
(16777327 . org-ref-cite-hydra/body) (follow-link . mouse-face)
(mouse-3 . org-find-file-at-mouse) (mouse-2 . org-open-at-mouse))
)
("citeauthor*" :follow
(lambda (_) (funcall org-ref-cite-onclick-function nil)) :export
org-ref-format-citeauthor* :complete org-citeauthor*-complete-link
:help-echo
(lambda (window object position)
(when org-ref-show-citation-on-enter
(save-excursion (goto-char position)
(let
((s (org-ref-format-entry (org-ref-get-bibtex-key-under-cursor)))
)
(with-temp-buffer (insert s) (fill-paragraph) (buffer-string)))
)
)
)
:face org-ref-cite-link-face-fn :display full :keymap
(keymap
(tab lambda nil (interactive)
(funcall org-ref-insert-cite-function))
(S-up . org-ref-sort-citation-link)
(S-right lambda nil (interactive) (org-ref-swap-citation-link 1))
(S-left lambda nil (interactive) (org-ref-swap-citation-link -1))
(C-right . org-ref-next-key) (C-left . org-ref-previous-key)
(16777337 lambda nil
"Paste key at point. Assumes the first thing in the killring is a key."
(interactive) (org-ref-insert-key-at-point (car kill-ring)))
(16777303 lambda nil "Copy all the keys at point." (interactive)
(kill-new (org-element-property :path (org-element-context))))
(16777335 lambda nil (interactive)
(kill-new (car (org-ref-get-bibtex-key-and-file))))
(16777318 lambda nil (interactive)
(save-excursion (org-ref-open-citation-at-point)
(kill-new (org-ref-format-bibtex-entry-at-point)))
)
(16777319 . org-ref-google-scholar-at-point)
(16777317 lambda nil "Email entry at point" (interactive)
(org-ref-open-citation-at-point) (org-ref-email-bibtex-entry))
(16777315 . org-ref-wos-citing-at-point)
(16777330 . org-ref-wos-related-at-point)
(16777326 . org-ref-open-notes-at-point)
(16777328 . org-ref-open-pdf-at-point)
(16777333 . org-ref-open-url-at-point)
(16777314 . org-ref-open-citation-at-point)
(16777327 . org-ref-cite-hydra/body) (follow-link . mouse-face)
(mouse-3 . org-find-file-at-mouse) (mouse-2 . org-open-at-mouse))
)
("citeauthor" :follow
(lambda (_) (funcall org-ref-cite-onclick-function nil)) :export
org-ref-format-citeauthor :complete org-citeauthor-complete-link
:help-echo
(lambda (window object position)
(when org-ref-show-citation-on-enter
(save-excursion (goto-char position)
(let
((s (org-ref-format-entry (org-ref-get-bibtex-key-under-cursor)))
)
(with-temp-buffer (insert s) (fill-paragraph) (buffer-string)))
)
)
)
:face org-ref-cite-link-face-fn :display full :keymap
(keymap
(tab lambda nil (interactive)
(funcall org-ref-insert-cite-function))
(S-up . org-ref-sort-citation-link)
(S-right lambda nil (interactive) (org-ref-swap-citation-link 1))
(S-left lambda nil (interactive) (org-ref-swap-citation-link -1))
(C-right . org-ref-next-key) (C-left . org-ref-previous-key)
(16777337 lambda nil
"Paste key at point. Assumes the first thing in the killring is a key."
(interactive) (org-ref-insert-key-at-point (car kill-ring)))
(16777303 lambda nil "Copy all the keys at point." (interactive)
(kill-new (org-element-property :path (org-element-context))))
(16777335 lambda nil (interactive)
(kill-new (car (org-ref-get-bibtex-key-and-file))))
(16777318 lambda nil (interactive)
(save-excursion (org-ref-open-citation-at-point)
(kill-new (org-ref-format-bibtex-entry-at-point)))
)
(16777319 . org-ref-google-scholar-at-point)
(16777317 lambda nil "Email entry at point" (interactive)
(org-ref-open-citation-at-point) (org-ref-email-bibtex-entry))
(16777315 . org-ref-wos-citing-at-point)
(16777330 . org-ref-wos-related-at-point)
(16777326 . org-ref-open-notes-at-point)
(16777328 . org-ref-open-pdf-at-point)
(16777333 . org-ref-open-url-at-point)
(16777314 . org-ref-open-citation-at-point)
(16777327 . org-ref-cite-hydra/body) (follow-link . mouse-face)
(mouse-3 . org-find-file-at-mouse) (mouse-2 . org-open-at-mouse))
)
("citetext" :follow
(lambda (_) (funcall org-ref-cite-onclick-function nil)) :export
org-ref-format-citetext :complete org-citetext-complete-link
:help-echo
(lambda (window object position)
(when org-ref-show-citation-on-enter
(save-excursion (goto-char position)
(let
((s (org-ref-format-entry (org-ref-get-bibtex-key-under-cursor)))
)
(with-temp-buffer (insert s) (fill-paragraph) (buffer-string)))
)
)
)
:face org-ref-cite-link-face-fn :display full :keymap
(keymap
(tab lambda nil (interactive)
(funcall org-ref-insert-cite-function))
(S-up . org-ref-sort-citation-link)
(S-right lambda nil (interactive) (org-ref-swap-citation-link 1))
(S-left lambda nil (interactive) (org-ref-swap-citation-link -1))
(C-right . org-ref-next-key) (C-left . org-ref-previous-key)
(16777337 lambda nil
"Paste key at point. Assumes the first thing in the killring is a key."
(interactive) (org-ref-insert-key-at-point (car kill-ring)))
(16777303 lambda nil "Copy all the keys at point." (interactive)
(kill-new (org-element-property :path (org-element-context))))
(16777335 lambda nil (interactive)
(kill-new (car (org-ref-get-bibtex-key-and-file))))
(16777318 lambda nil (interactive)
(save-excursion (org-ref-open-citation-at-point)
(kill-new (org-ref-format-bibtex-entry-at-point)))
)
(16777319 . org-ref-google-scholar-at-point)
(16777317 lambda nil "Email entry at point" (interactive)
(org-ref-open-citation-at-point) (org-ref-email-bibtex-entry))
(16777315 . org-ref-wos-citing-at-point)
(16777330 . org-ref-wos-related-at-point)
(16777326 . org-ref-open-notes-at-point)
(16777328 . org-ref-open-pdf-at-point)
(16777333 . org-ref-open-url-at-point)
(16777314 . org-ref-open-citation-at-point)
(16777327 . org-ref-cite-hydra/body) (follow-link . mouse-face)
(mouse-3 . org-find-file-at-mouse) (mouse-2 . org-open-at-mouse))
)
("citenum" :follow
(lambda (_) (funcall org-ref-cite-onclick-function nil)) :export
org-ref-format-citenum :complete org-citenum-complete-link
:help-echo
(lambda (window object position)
(when org-ref-show-citation-on-enter
(save-excursion (goto-char position)
(let
((s (org-ref-format-entry (org-ref-get-bibtex-key-under-cursor)))
)
(with-temp-buffer (insert s) (fill-paragraph) (buffer-string)))
)
)
)
:face org-ref-cite-link-face-fn :display full :keymap
(keymap
(tab lambda nil (interactive)
(funcall org-ref-insert-cite-function))
(S-up . org-ref-sort-citation-link)
(S-right lambda nil (interactive) (org-ref-swap-citation-link 1))
(S-left lambda nil (interactive) (org-ref-swap-citation-link -1))
(C-right . org-ref-next-key) (C-left . org-ref-previous-key)
(16777337 lambda nil
"Paste key at point. Assumes the first thing in the killring is a key."
(interactive) (org-ref-insert-key-at-point (car kill-ring)))
(16777303 lambda nil "Copy all the keys at point." (interactive)
(kill-new (org-element-property :path (org-element-context))))
(16777335 lambda nil (interactive)
(kill-new (car (org-ref-get-bibtex-key-and-file))))
(16777318 lambda nil (interactive)
(save-excursion (org-ref-open-citation-at-point)
(kill-new (org-ref-format-bibtex-entry-at-point)))
)
(16777319 . org-ref-google-scholar-at-point)
(16777317 lambda nil "Email entry at point" (interactive)
(org-ref-open-citation-at-point) (org-ref-email-bibtex-entry))
(16777315 . org-ref-wos-citing-at-point)
(16777330 . org-ref-wos-related-at-point)
(16777326 . org-ref-open-notes-at-point)
(16777328 . org-ref-open-pdf-at-point)
(16777333 . org-ref-open-url-at-point)
(16777314 . org-ref-open-citation-at-point)
(16777327 . org-ref-cite-hydra/body) (follow-link . mouse-face)
(mouse-3 . org-find-file-at-mouse) (mouse-2 . org-open-at-mouse))
)
("citealp*" :follow
(lambda (_) (funcall org-ref-cite-onclick-function nil)) :export
org-ref-format-citealp* :complete org-citealp*-complete-link
:help-echo
(lambda (window object position)
(when org-ref-show-citation-on-enter
(save-excursion (goto-char position)
(let
((s (org-ref-format-entry (org-ref-get-bibtex-key-under-cursor)))
)
(with-temp-buffer (insert s) (fill-paragraph) (buffer-string)))
)
)
)
:face org-ref-cite-link-face-fn :display full :keymap
(keymap
(tab lambda nil (interactive)
(funcall org-ref-insert-cite-function))
(S-up . org-ref-sort-citation-link)
(S-right lambda nil (interactive) (org-ref-swap-citation-link 1))
(S-left lambda nil (interactive) (org-ref-swap-citation-link -1))
(C-right . org-ref-next-key) (C-left . org-ref-previous-key)
(16777337 lambda nil
"Paste key at point. Assumes the first thing in the killring is a key."
(interactive) (org-ref-insert-key-at-point (car kill-ring)))
(16777303 lambda nil "Copy all the keys at point." (interactive)
(kill-new (org-element-property :path (org-element-context))))
(16777335 lambda nil (interactive)
(kill-new (car (org-ref-get-bibtex-key-and-file))))
(16777318 lambda nil (interactive)
(save-excursion (org-ref-open-citation-at-point)
(kill-new (org-ref-format-bibtex-entry-at-point)))
)
(16777319 . org-ref-google-scholar-at-point)
(16777317 lambda nil "Email entry at point" (interactive)
(org-ref-open-citation-at-point) (org-ref-email-bibtex-entry))
(16777315 . org-ref-wos-citing-at-point)
(16777330 . org-ref-wos-related-at-point)
(16777326 . org-ref-open-notes-at-point)
(16777328 . org-ref-open-pdf-at-point)
(16777333 . org-ref-open-url-at-point)
(16777314 . org-ref-open-citation-at-point)
(16777327 . org-ref-cite-hydra/body) (follow-link . mouse-face)
(mouse-3 . org-find-file-at-mouse) (mouse-2 . org-open-at-mouse))
)
("citealp" :follow
(lambda (_) (funcall org-ref-cite-onclick-function nil)) :export
org-ref-format-citealp :complete org-citealp-complete-link
:help-echo
(lambda (window object position)
(when org-ref-show-citation-on-enter
(save-excursion (goto-char position)
(let
((s (org-ref-format-entry (org-ref-get-bibtex-key-under-cursor)))
)
(with-temp-buffer (insert s) (fill-paragraph) (buffer-string)))
)
)
)
:face org-ref-cite-link-face-fn :display full :keymap
(keymap
(tab lambda nil (interactive)
(funcall org-ref-insert-cite-function))
(S-up . org-ref-sort-citation-link)
(S-right lambda nil (interactive) (org-ref-swap-citation-link 1))
(S-left lambda nil (interactive) (org-ref-swap-citation-link -1))
(C-right . org-ref-next-key) (C-left . org-ref-previous-key)
(16777337 lambda nil
"Paste key at point. Assumes the first thing in the killring is a key."
(interactive) (org-ref-insert-key-at-point (car kill-ring)))
(16777303 lambda nil "Copy all the keys at point." (interactive)
(kill-new (org-element-property :path (org-element-context))))
(16777335 lambda nil (interactive)
(kill-new (car (org-ref-get-bibtex-key-and-file))))
(16777318 lambda nil (interactive)
(save-excursion (org-ref-open-citation-at-point)
(kill-new (org-ref-format-bibtex-entry-at-point)))
)
(16777319 . org-ref-google-scholar-at-point)
(16777317 lambda nil "Email entry at point" (interactive)
(org-ref-open-citation-at-point) (org-ref-email-bibtex-entry))
(16777315 . org-ref-wos-citing-at-point)
(16777330 . org-ref-wos-related-at-point)
(16777326 . org-ref-open-notes-at-point)
(16777328 . org-ref-open-pdf-at-point)
(16777333 . org-ref-open-url-at-point)
(16777314 . org-ref-open-citation-at-point)
(16777327 . org-ref-cite-hydra/body) (follow-link . mouse-face)
(mouse-3 . org-find-file-at-mouse) (mouse-2 . org-open-at-mouse))
)
("citealt*" :follow
(lambda (_) (funcall org-ref-cite-onclick-function nil)) :export
org-ref-format-citealt* :complete org-citealt*-complete-link
:help-echo
(lambda (window object position)
(when org-ref-show-citation-on-enter
(save-excursion (goto-char position)
(let
((s (org-ref-format-entry (org-ref-get-bibtex-key-under-cursor)))
)
(with-temp-buffer (insert s) (fill-paragraph) (buffer-string)))
)
)
)
:face org-ref-cite-link-face-fn :display full :keymap
(keymap
(tab lambda nil (interactive)
(funcall org-ref-insert-cite-function))
(S-up . org-ref-sort-citation-link)
(S-right lambda nil (interactive) (org-ref-swap-citation-link 1))
(S-left lambda nil (interactive) (org-ref-swap-citation-link -1))
(C-right . org-ref-next-key) (C-left . org-ref-previous-key)
(16777337 lambda nil
"Paste key at point. Assumes the first thing in the killring is a key."
(interactive) (org-ref-insert-key-at-point (car kill-ring)))
(16777303 lambda nil "Copy all the keys at point." (interactive)
(kill-new (org-element-property :path (org-element-context))))
(16777335 lambda nil (interactive)
(kill-new (car (org-ref-get-bibtex-key-and-file))))
(16777318 lambda nil (interactive)
(save-excursion (org-ref-open-citation-at-point)
(kill-new (org-ref-format-bibtex-entry-at-point)))
)
(16777319 . org-ref-google-scholar-at-point)
(16777317 lambda nil "Email entry at point" (interactive)
(org-ref-open-citation-at-point) (org-ref-email-bibtex-entry))
(16777315 . org-ref-wos-citing-at-point)
(16777330 . org-ref-wos-related-at-point)
(16777326 . org-ref-open-notes-at-point)
(16777328 . org-ref-open-pdf-at-point)
(16777333 . org-ref-open-url-at-point)
(16777314 . org-ref-open-citation-at-point)
(16777327 . org-ref-cite-hydra/body) (follow-link . mouse-face)
(mouse-3 . org-find-file-at-mouse) (mouse-2 . org-open-at-mouse))
)
("citealt" :follow
(lambda (_) (funcall org-ref-cite-onclick-function nil)) :export
org-ref-format-citealt :complete org-citealt-complete-link
:help-echo
(lambda (window object position)
(when org-ref-show-citation-on-enter
(save-excursion (goto-char position)
(let
((s (org-ref-format-entry (org-ref-get-bibtex-key-under-cursor)))
)
(with-temp-buffer (insert s) (fill-paragraph) (buffer-string)))
)
)
)
:face org-ref-cite-link-face-fn :display full :keymap
(keymap
(tab lambda nil (interactive)
(funcall org-ref-insert-cite-function))
(S-up . org-ref-sort-citation-link)
(S-right lambda nil (interactive) (org-ref-swap-citation-link 1))
(S-left lambda nil (interactive) (org-ref-swap-citation-link -1))
(C-right . org-ref-next-key) (C-left . org-ref-previous-key)
(16777337 lambda nil
"Paste key at point. Assumes the first thing in the killring is a key."
(interactive) (org-ref-insert-key-at-point (car kill-ring)))
(16777303 lambda nil "Copy all the keys at point." (interactive)
(kill-new (org-element-property :path (org-element-context))))
(16777335 lambda nil (interactive)
(kill-new (car (org-ref-get-bibtex-key-and-file))))
(16777318 lambda nil (interactive)
(save-excursion (org-ref-open-citation-at-point)
(kill-new (org-ref-format-bibtex-entry-at-point)))
)
(16777319 . org-ref-google-scholar-at-point)
(16777317 lambda nil "Email entry at point" (interactive)
(org-ref-open-citation-at-point) (org-ref-email-bibtex-entry))
(16777315 . org-ref-wos-citing-at-point)
(16777330 . org-ref-wos-related-at-point)
(16777326 . org-ref-open-notes-at-point)
(16777328 . org-ref-open-pdf-at-point)
(16777333 . org-ref-open-url-at-point)
(16777314 . org-ref-open-citation-at-point)
(16777327 . org-ref-cite-hydra/body) (follow-link . mouse-face)
(mouse-3 . org-find-file-at-mouse) (mouse-2 . org-open-at-mouse))
)
("citep*" :follow
(lambda (_) (funcall org-ref-cite-onclick-function nil)) :export
org-ref-format-citep* :complete org-citep*-complete-link :help-echo
(lambda (window object position)
(when org-ref-show-citation-on-enter
(save-excursion (goto-char position)
(let
((s (org-ref-format-entry (org-ref-get-bibtex-key-under-cursor)))
)
(with-temp-buffer (insert s) (fill-paragraph) (buffer-string)))
)
)
)
:face org-ref-cite-link-face-fn :display full :keymap
(keymap
(tab lambda nil (interactive)
(funcall org-ref-insert-cite-function))
(S-up . org-ref-sort-citation-link)
(S-right lambda nil (interactive) (org-ref-swap-citation-link 1))
(S-left lambda nil (interactive) (org-ref-swap-citation-link -1))
(C-right . org-ref-next-key) (C-left . org-ref-previous-key)
(16777337 lambda nil
"Paste key at point. Assumes the first thing in the killring is a key."
(interactive) (org-ref-insert-key-at-point (car kill-ring)))
(16777303 lambda nil "Copy all the keys at point." (interactive)
(kill-new (org-element-property :path (org-element-context))))
(16777335 lambda nil (interactive)
(kill-new (car (org-ref-get-bibtex-key-and-file))))
(16777318 lambda nil (interactive)
(save-excursion (org-ref-open-citation-at-point)
(kill-new (org-ref-format-bibtex-entry-at-point)))
)
(16777319 . org-ref-google-scholar-at-point)
(16777317 lambda nil "Email entry at point" (interactive)
(org-ref-open-citation-at-point) (org-ref-email-bibtex-entry))
(16777315 . org-ref-wos-citing-at-point)
(16777330 . org-ref-wos-related-at-point)
(16777326 . org-ref-open-notes-at-point)
(16777328 . org-ref-open-pdf-at-point)
(16777333 . org-ref-open-url-at-point)
(16777314 . org-ref-open-citation-at-point)
(16777327 . org-ref-cite-hydra/body) (follow-link . mouse-face)
(mouse-3 . org-find-file-at-mouse) (mouse-2 . org-open-at-mouse))
)
("citep" :follow
(lambda (_) (funcall org-ref-cite-onclick-function nil)) :export
org-ref-format-citep :complete org-citep-complete-link :help-echo
(lambda (window object position)
(when org-ref-show-citation-on-enter
(save-excursion (goto-char position)
(let
((s (org-ref-format-entry (org-ref-get-bibtex-key-under-cursor)))
)
(with-temp-buffer (insert s) (fill-paragraph) (buffer-string)))
)
)
)
:face org-ref-cite-link-face-fn :display full :keymap
(keymap
(tab lambda nil (interactive)
(funcall org-ref-insert-cite-function))
(S-up . org-ref-sort-citation-link)
(S-right lambda nil (interactive) (org-ref-swap-citation-link 1))
(S-left lambda nil (interactive) (org-ref-swap-citation-link -1))
(C-right . org-ref-next-key) (C-left . org-ref-previous-key)
(16777337 lambda nil
"Paste key at point. Assumes the first thing in the killring is a key."
(interactive) (org-ref-insert-key-at-point (car kill-ring)))
(16777303 lambda nil "Copy all the keys at point." (interactive)
(kill-new (org-element-property :path (org-element-context))))
(16777335 lambda nil (interactive)
(kill-new (car (org-ref-get-bibtex-key-and-file))))
(16777318 lambda nil (interactive)
(save-excursion (org-ref-open-citation-at-point)
(kill-new (org-ref-format-bibtex-entry-at-point)))
)
(16777319 . org-ref-google-scholar-at-point)
(16777317 lambda nil "Email entry at point" (interactive)
(org-ref-open-citation-at-point) (org-ref-email-bibtex-entry))
(16777315 . org-ref-wos-citing-at-point)
(16777330 . org-ref-wos-related-at-point)
(16777326 . org-ref-open-notes-at-point)
(16777328 . org-ref-open-pdf-at-point)
(16777333 . org-ref-open-url-at-point)
(16777314 . org-ref-open-citation-at-point)
(16777327 . org-ref-cite-hydra/body) (follow-link . mouse-face)
(mouse-3 . org-find-file-at-mouse) (mouse-2 . org-open-at-mouse))
)
("citet*" :follow
(lambda (_) (funcall org-ref-cite-onclick-function nil)) :export
org-ref-format-citet* :complete org-citet*-complete-link :help-echo
(lambda (window object position)
(when org-ref-show-citation-on-enter
(save-excursion (goto-char position)
(let
((s (org-ref-format-entry (org-ref-get-bibtex-key-under-cursor)))
)
(with-temp-buffer (insert s) (fill-paragraph) (buffer-string)))
)
)
)
:face org-ref-cite-link-face-fn :display full :keymap
(keymap
(tab lambda nil (interactive)
(funcall org-ref-insert-cite-function))
(S-up . org-ref-sort-citation-link)
(S-right lambda nil (interactive) (org-ref-swap-citation-link 1))
(S-left lambda nil (interactive) (org-ref-swap-citation-link -1))
(C-right . org-ref-next-key) (C-left . org-ref-previous-key)
(16777337 lambda nil
"Paste key at point. Assumes the first thing in the killring is a key."
(interactive) (org-ref-insert-key-at-point (car kill-ring)))
(16777303 lambda nil "Copy all the keys at point." (interactive)
(kill-new (org-element-property :path (org-element-context))))
(16777335 lambda nil (interactive)
(kill-new (car (org-ref-get-bibtex-key-and-file))))
(16777318 lambda nil (interactive)
(save-excursion (org-ref-open-citation-at-point)
(kill-new (org-ref-format-bibtex-entry-at-point)))
)
(16777319 . org-ref-google-scholar-at-point)
(16777317 lambda nil "Email entry at point" (interactive)
(org-ref-open-citation-at-point) (org-ref-email-bibtex-entry))
(16777315 . org-ref-wos-citing-at-point)
(16777330 . org-ref-wos-related-at-point)
(16777326 . org-ref-open-notes-at-point)
(16777328 . org-ref-open-pdf-at-point)
(16777333 . org-ref-open-url-at-point)
(16777314 . org-ref-open-citation-at-point)
(16777327 . org-ref-cite-hydra/body) (follow-link . mouse-face)
(mouse-3 . org-find-file-at-mouse) (mouse-2 . org-open-at-mouse))
)
("citet" :follow
(lambda (_) (funcall org-ref-cite-onclick-function nil)) :export
org-ref-format-citet :complete org-citet-complete-link :help-echo
(lambda (window object position)
(when org-ref-show-citation-on-enter
(save-excursion (goto-char position)
(let
((s (org-ref-format-entry (org-ref-get-bibtex-key-under-cursor)))
)
(with-temp-buffer (insert s) (fill-paragraph) (buffer-string)))
)
)
)
:face org-ref-cite-link-face-fn :display full :keymap
(keymap
(tab lambda nil (interactive)
(funcall org-ref-insert-cite-function))
(S-up . org-ref-sort-citation-link)
(S-right lambda nil (interactive) (org-ref-swap-citation-link 1))
(S-left lambda nil (interactive) (org-ref-swap-citation-link -1))
(C-right . org-ref-next-key) (C-left . org-ref-previous-key)
(16777337 lambda nil
"Paste key at point. Assumes the first thing in the killring is a key."
(interactive) (org-ref-insert-key-at-point (car kill-ring)))
(16777303 lambda nil "Copy all the keys at point." (interactive)
(kill-new (org-element-property :path (org-element-context))))
(16777335 lambda nil (interactive)
(kill-new (car (org-ref-get-bibtex-key-and-file))))
(16777318 lambda nil (interactive)
(save-excursion (org-ref-open-citation-at-point)
(kill-new (org-ref-format-bibtex-entry-at-point)))
)
(16777319 . org-ref-google-scholar-at-point)
(16777317 lambda nil "Email entry at point" (interactive)
(org-ref-open-citation-at-point) (org-ref-email-bibtex-entry))
(16777315 . org-ref-wos-citing-at-point)
(16777330 . org-ref-wos-related-at-point)
(16777326 . org-ref-open-notes-at-point)
(16777328 . org-ref-open-pdf-at-point)
(16777333 . org-ref-open-url-at-point)
(16777314 . org-ref-open-citation-at-point)
(16777327 . org-ref-cite-hydra/body) (follow-link . mouse-face)
(mouse-3 . org-find-file-at-mouse) (mouse-2 . org-open-at-mouse))
)
("nocite" :follow
(lambda (_) (funcall org-ref-cite-onclick-function nil)) :export
org-ref-format-nocite :complete org-nocite-complete-link :help-echo
(lambda (window object position)
(when org-ref-show-citation-on-enter
(save-excursion (goto-char position)
(let
((s (org-ref-format-entry (org-ref-get-bibtex-key-under-cursor)))
)
(with-temp-buffer (insert s) (fill-paragraph) (buffer-string)))
)
)
)
:face org-ref-cite-link-face-fn :display full :keymap
(keymap
(tab lambda nil (interactive)
(funcall org-ref-insert-cite-function))
(S-up . org-ref-sort-citation-link)
(S-right lambda nil (interactive) (org-ref-swap-citation-link 1))
(S-left lambda nil (interactive) (org-ref-swap-citation-link -1))
(C-right . org-ref-next-key) (C-left . org-ref-previous-key)
(16777337 lambda nil
"Paste key at point. Assumes the first thing in the killring is a key."
(interactive) (org-ref-insert-key-at-point (car kill-ring)))
(16777303 lambda nil "Copy all the keys at point." (interactive)
(kill-new (org-element-property :path (org-element-context))))
(16777335 lambda nil (interactive)
(kill-new (car (org-ref-get-bibtex-key-and-file))))
(16777318 lambda nil (interactive)
(save-excursion (org-ref-open-citation-at-point)
(kill-new (org-ref-format-bibtex-entry-at-point)))
)
(16777319 . org-ref-google-scholar-at-point)
(16777317 lambda nil "Email entry at point" (interactive)
(org-ref-open-citation-at-point) (org-ref-email-bibtex-entry))
(16777315 . org-ref-wos-citing-at-point)
(16777330 . org-ref-wos-related-at-point)
(16777326 . org-ref-open-notes-at-point)
(16777328 . org-ref-open-pdf-at-point)
(16777333 . org-ref-open-url-at-point)
(16777314 . org-ref-open-citation-at-point)
(16777327 . org-ref-cite-hydra/body) (follow-link . mouse-face)
(mouse-3 . org-find-file-at-mouse) (mouse-2 . org-open-at-mouse))
)
("cite" :follow
(lambda (_) (funcall org-ref-cite-onclick-function nil)) :export
org-ref-format-cite :complete org-cite-complete-link :help-echo
(lambda (window object position)
(when org-ref-show-citation-on-enter
(save-excursion (goto-char position)
(let
((s (org-ref-format-entry (org-ref-get-bibtex-key-under-cursor)))
)
(with-temp-buffer (insert s) (fill-paragraph) (buffer-string)))
)
)
)
:face org-ref-cite-link-face-fn :display full :keymap
(keymap
(tab lambda nil (interactive)
(funcall org-ref-insert-cite-function))
(S-up . org-ref-sort-citation-link)
(S-right lambda nil (interactive) (org-ref-swap-citation-link 1))
(S-left lambda nil (interactive) (org-ref-swap-citation-link -1))
(C-right . org-ref-next-key) (C-left . org-ref-previous-key)
(16777337 lambda nil
"Paste key at point. Assumes the first thing in the killring is a key."
(interactive) (org-ref-insert-key-at-point (car kill-ring)))
(16777303 lambda nil "Copy all the keys at point." (interactive)
(kill-new (org-element-property :path (org-element-context))))
(16777335 lambda nil (interactive)
(kill-new (car (org-ref-get-bibtex-key-and-file))))
(16777318 lambda nil (interactive)
(save-excursion (org-ref-open-citation-at-point)
(kill-new (org-ref-format-bibtex-entry-at-point)))
)
(16777319 . org-ref-google-scholar-at-point)
(16777317 lambda nil "Email entry at point" (interactive)
(org-ref-open-citation-at-point) (org-ref-email-bibtex-entry))
(16777315 . org-ref-wos-citing-at-point)
(16777330 . org-ref-wos-related-at-point)
(16777326 . org-ref-open-notes-at-point)
(16777328 . org-ref-open-pdf-at-point)
(16777333 . org-ref-open-url-at-point)
(16777314 . org-ref-open-citation-at-point)
(16777327 . org-ref-cite-hydra/body) (follow-link . mouse-face)
(mouse-3 . org-find-file-at-mouse) (mouse-2 . org-open-at-mouse))
:store org-ref-bibtex-store-link)
("Cref" :follow org-ref-ref-follow :export org-ref-Cref-export
:complete org-ref-complete-link :face org-ref-ref-face-fn :help-echo
org-ref-ref-help-echo)
("cref" :follow org-ref-ref-follow :export org-ref-cref-export
:complete org-ref-complete-link :face org-ref-ref-face-fn :help-echo
org-ref-ref-help-echo)
("autoref" :follow org-ref-ref-follow :export org-ref-autoref-export
:complete org-ref-complete-link :face org-ref-ref-face-fn :help-echo
org-ref-ref-help-echo)
("eqref" :follow org-ref-ref-follow :export org-ref-eqref-export
:complete org-ref-complete-link :face org-ref-ref-face-fn :help-echo
org-ref-ref-help-echo)
("nameref" :follow org-ref-ref-follow :export org-ref-export-nameref
:complete org-ref-complete-link :face org-ref-ref-face-fn :help-echo
org-ref-ref-help-echo)
("pageref" :follow org-ref-ref-follow :export
#[(path desc format)
"\b\302\267\202\300\303 \"\207\300\304 \"\207\305\207"
[format path #s
(hash-table size 2 test eq rehash-size 1.5 rehash-threshold
0.8125 purecopy t data (html 6 latex 11))
"(<pageref>%s</pageref>)" "\\pageref{%s}" nil]
3]
:face org-ref-ref-face-fn :complete org-pageref-complete-link
:help-echo org-ref-ref-help-echo)
("ref" :follow org-ref-ref-follow :export org-ref-ref-export
:complete org-ref-complete-link :face org-ref-ref-face-fn :help-echo
org-ref-ref-help-echo)
("label" :follow
#[(label)
"\302\b!\303\304\305 \211\306U\204 \307V\203\310\202\311#\302\b!\")\207"
[label count org-ref-count-labels message format "%s occurence%s"
0 1 "s" ""]
6
"On clicking count the number of label tags used in the buffer.\nA 
number greater than one means multiple labels!"]
:export
#[(keyword desc format)
"\b\302\267\202\300\303 \"\207\300\304 \"\207\300\305 \"\207\306\207"
[format

keyword

#s

(hash-table




size




3




test




eq




rehash-size




1.5




rehash-threshold




0.8125




purecopy




t




data




(html 6 md 11 latex 16)




)

"<div id=\"%s\"></div>" "<a name=\"%s\"></a>" "\\label{%s}" nil]
3]
:store org-label-store-link :face org-ref-label-face-fn :help-echo
#[(window object position)
"\212\bb\210\303 \304\305!r\nq\210\306\216 c\210\307 \210\310 -\207"
[position

s

temp-buffer

org-ref-link-message

generate-new-buffer

" *temp*"

#[nil







"\301\b!\205 \302\b!\207"







[temp-buffer buffer-name kill-buffer]












2]

fill-paragraph buffer-string]
2]
)
("list-of-tables" :follow org-ref-list-of-tables :export
#[(keyword desc format) "\b\301=\205 \300\302!\207"
[format latex "\\listoftables"] 2]
)
("list-of-figures" :follow org-ref-list-of-figures :export
#[(keyword desc format) "\b\301=\205 \300\302!\207"
[format latex "\\listoffigures"] 2]
)
("addbibresource" :follow org-ref-follow-addbibresource :export
#[(keyword desc format)
"\b\302\267\202\300\303!\207\300\304 \"\207\305\207"
[format keyword #s
(hash-table size 2 test eq rehash-size 1.5 rehash-threshold
0.8125 purecopy t data (html 6 latex 10))
"" "\\addbibresource{%s}" nil]
3]
)
("bibliographystyle" :export
#[(keyword desc format)
"\b\302=\204\f\b\303=\203\300\304 \"\207\305\207"
[format keyword latex beamer "\\bibliographystyle{%s}" ""] 3]
)
("printbibliography" :follow org-ref-open-bibliography :export
#[(keyword desc format)
"\b\302\267\202\303 \207\304 \207 \207\305\207"
[format org-ref-printbibliography-cmd #s
(hash-table size 3 test eq rehash-size 1.5 rehash-threshold
0.8125 purecopy t data (org 6 html 9 latex 12))
org-ref-get-org-bibliography org-ref-get-html-bibliography nil]
2]
)
("nobibliography" :follow org-ref-open-bibliography :export
org-ref-nobibliography-format)
("bibliography" :follow org-ref-open-bibliography :export
org-ref-bibliography-format :complete org-bibliography-complete-link
:help-echo
#[(window object position)
"\212\bb\210\303 \304\305!r\nq\210\306\216 c\210\307 \210\310 -\207"
[position

s

temp-buffer

org-ref-link-message

generate-new-buffer

" *temp*"

#[nil







"\301\b!\205 \302\b!\207"







[temp-buffer buffer-name kill-buffer]












2]

fill-paragraph buffer-string]
2]
:face org-ref-bibliography-face-fn)
("Acp" :follow or-follow-acronym :face org-ref-acronym-face
:help-echo or-acronym-tooltip :export
#[771 "\211\301>\203\302\303\300A#\207\302\304\226\"\207"
[("Acp" . "Glspl") (latex beamer) format "\\%s{%s}" "%s"] 7
"\n\n(fn PATH _ FORMAT)"]
)
("acp" :follow or-follow-acronym :face org-ref-acronym-face
:help-echo or-acronym-tooltip :export
#[771 "\211\301>\203\302\303\300A#\207\302\304\226\"\207"
[("acp" . "glspl") (latex beamer) format "\\%s{%s}" "%s"] 7
"\n\n(fn PATH _ FORMAT)"]
)
("Ac" :follow or-follow-acronym :face org-ref-acronym-face :help-echo
or-acronym-tooltip :export
#[771 "\211\301>\203\302\303\300A#\207\302\304\226\"\207"
[("Ac" . "Gls") (latex beamer) format "\\%s{%s}" "%s"] 7
"\n\n(fn PATH _ FORMAT)"]
)
("ac" :follow or-follow-acronym :face org-ref-acronym-face :help-echo
or-acronym-tooltip :export
#[771 "\211\301>\203\302\303\300A#\207\302\304\226\"\207"
[("ac" . "gls") (latex beamer) format "\\%s{%s}" "%s"] 7
"\n\n(fn PATH _ FORMAT)"]
)
("acrfull" :follow or-follow-acronym :face org-ref-acronym-face
:help-echo or-acronym-tooltip :export
#[771 "\211\301>\203\302\303\300A#\207\302\304\226\"\207"
[("acrfull" . "acrfull") (latex beamer) format "\\%s{%s}" "%s"] 7
"\n\n(fn PATH _ FORMAT)"]
)
("acrlong" :follow or-follow-acronym :face org-ref-acronym-face
:help-echo or-acronym-tooltip :export
#[771 "\211\301>\203\302\303\300A#\207\302\304\226\"\207"
[("acrlong" . "acrlong") (latex beamer) format "\\%s{%s}" "%s"] 7
"\n\n(fn PATH _ FORMAT)"]
)
("acrshort" :follow or-follow-acronym :face org-ref-acronym-face
:help-echo or-acronym-tooltip :export
#[771 "\211\301>\203\302\303\300A#\207\302\304\226\"\207"
[("acrshort" . "acrshort") (latex beamer) format "\\%s{%s}" "%s"]
7 "\n\n(fn PATH _ FORMAT)"]
)
("glslink" :follow or-follow-glossary :face org-ref-glossary-face
:help-echo or-glossary-tooltip :export
#[771 "\211\300>\203\f\301\302\x04#\207\301\303\"\207"
[(latex beamer) format "\\glslink{%s}{%s}" "%s"] 7
"\n\n(fn PATH DESC FORMAT)"]
)
("glsdesc" :follow or-follow-glossary :face org-ref-glossary-face
:help-echo or-glossary-tooltip :export
#[771 "\211\301>\203\f\302\303\300#\207\302\304\"\207"
["glsdesc" (latex beamer) format "\\%s{%s}" "%s"] 7
"\n\n(fn PATH _ FORMAT)"]
)
("glssymbol" :follow or-follow-glossary :face org-ref-glossary-face
:help-echo or-glossary-tooltip :export
#[771 "\211\301>\203\f\302\303\300#\207\302\304\"\207"
["glssymbol" (latex beamer) format "\\%s{%s}" "%s"] 7
"\n\n(fn PATH _ FORMAT)"]
)
("Glspl" :follow or-follow-glossary :face org-ref-glossary-face
:help-echo or-glossary-tooltip :export
#[771 "\211\301>\203\f\302\303\300#\207\302\304\"\207"
["Glspl" (latex beamer) format "\\%s{%s}" "%s"] 7
"\n\n(fn PATH _ FORMAT)"]
)
("Gls" :follow or-follow-glossary :face org-ref-glossary-face
:help-echo or-glossary-tooltip :export
#[771 "\211\301>\203\f\302\303\300#\207\302\304\"\207"
["Gls" (latex beamer) format "\\%s{%s}" "%s"] 7
"\n\n(fn PATH _ FORMAT)"]
)
("glspl" :follow or-follow-glossary :face org-ref-glossary-face
:help-echo or-glossary-tooltip :export
#[771 "\211\301>\203\f\302\303\300#\207\302\304\"\207"
["glspl" (latex beamer) format "\\%s{%s}" "%s"] 7
"\n\n(fn PATH _ FORMAT)"]
)
("gls" :follow or-follow-glossary :face org-ref-glossary-face
:help-echo or-glossary-tooltip :export
#[771 "\211\301>\203\f\302\303\300#\207\302\304\"\207"
["gls" (latex beamer) format "\\%s{%s}" "%s"] 7
"\n\n(fn PATH _ FORMAT)"]
)
("bibtex" :follow org-bibtex-open :store org-bibtex-store-link)
("image" :export org-roam-server-export-image-id)
("server" :export org-roam-server-export-server-id)
("pdf" :follow org-pdftools-open :complete org-pdftools-complete-link
:store org-pdftools-store-link :export org-pdftools-export)
("orgit-rev" :store orgit-rev-store :follow orgit-rev-open :export
orgit-rev-export :complete orgit-rev-complete-link)
("orgit-log" :store orgit-log-store :follow orgit-log-open :export
orgit-log-export :complete orgit-log-complete-link)
("orgit" :store orgit-status-store :follow orgit-status-open :export
orgit-status-export :complete orgit-status-complete-link)
("roam" :follow org-roam-link-follow-link)
("id" :follow org-id-open :face org-roam--id-link-face)
("yt" :follow org-yt-follow :image-data-fun org-yt-image-data-fun)
("img" :image-data-fun +org-inline-image-data-fn)
("doom-modules" :complete
(closure
((dir-fn closure
((plist) (dir-var . doom-modules-dir) (key . "doom-modules") t)
nil (symbol-value dir-var))
(requires) (plist) (dir-var . doom-modules-dir)
(key . "doom-modules") t)
nil (if requires (mapc #'require (doom-enlist requires)))
(+org--relative-path (+org--read-link-path key (funcall dir-fn))
(funcall dir-fn))
)
:follow
(closure
((dir-fn closure
((plist) (dir-var . doom-modules-dir) (key . "doom-modules") t)
nil (symbol-value dir-var))
(requires) (plist) (dir-var . doom-modules-dir)
(key . "doom-modules") t)
(link)
(org-link-open-as-file (expand-file-name link (funcall dir-fn)) nil)
)
:face
(closure
((dir-fn closure
((plist) (dir-var . doom-modules-dir) (key . "doom-modules") t)
nil (symbol-value dir-var))
(requires) (plist) (dir-var . doom-modules-dir)
(key . "doom-modules") t)
(link)
(let*
((path (expand-file-name link (funcall dir-fn)))
(option-index (string-match-p "::\\(.*\\)\\'" path))
(file-name (substring path 0 option-index)))
(if (file-exists-p file-name) 'org-link 'error))
)
)
("doom-docs" :complete
(closure
((dir-fn closure
((plist) (dir-var . doom-docs-dir) (key . "doom-docs") t) nil
(symbol-value dir-var))
(requires) (plist) (dir-var . doom-docs-dir) (key . "doom-docs") t)
nil (if requires (mapc #'require (doom-enlist requires)))
(+org--relative-path (+org--read-link-path key (funcall dir-fn))
(funcall dir-fn))
)
:follow
(closure
((dir-fn closure
((plist) (dir-var . doom-docs-dir) (key . "doom-docs") t) nil
(symbol-value dir-var))
(requires) (plist) (dir-var . doom-docs-dir) (key . "doom-docs") t)
(link)
(org-link-open-as-file (expand-file-name link (funcall dir-fn)) nil)
)
:face
(closure
((dir-fn closure
((plist) (dir-var . doom-docs-dir) (key . "doom-docs") t) nil
(symbol-value dir-var))
(requires) (plist) (dir-var . doom-docs-dir) (key . "doom-docs") t)
(link)
(let*
((path (expand-file-name link (funcall dir-fn)))
(option-index (string-match-p "::\\(.*\\)\\'" path))
(file-name (substring path 0 option-index)))
(if (file-exists-p file-name) 'org-link 'error))
)
)
("doom" :complete
(closure
((dir-fn closure
((plist) (dir-var . doom-emacs-dir) (key . "doom") t) nil
(symbol-value dir-var))
(requires) (plist) (dir-var . doom-emacs-dir) (key . "doom") t)
nil (if requires (mapc #'require (doom-enlist requires)))
(+org--relative-path (+org--read-link-path key (funcall dir-fn))
(funcall dir-fn))
)
:follow
(closure
((dir-fn closure
((plist) (dir-var . doom-emacs-dir) (key . "doom") t) nil
(symbol-value dir-var))
(requires) (plist) (dir-var . doom-emacs-dir) (key . "doom") t)
(link)
(org-link-open-as-file (expand-file-name link (funcall dir-fn)) nil)
)
:face
(closure
((dir-fn closure
((plist) (dir-var . doom-emacs-dir) (key . "doom") t) nil
(symbol-value dir-var))
(requires) (plist) (dir-var . doom-emacs-dir) (key . "doom") t)
(link)
(let*
((path (expand-file-name link (funcall dir-fn)))
(option-index (string-match-p "::\\(.*\\)\\'" path))
(file-name (substring path 0 option-index)))
(if (file-exists-p file-name) 'org-link 'error))
)
)
("org" :complete
(closure
((dir-fn closure
((plist) (dir-var . org-directory) (key . "org") t) nil
(symbol-value dir-var))
(requires) (plist) (dir-var . org-directory) (key . "org") t)
nil (if requires (mapc #'require (doom-enlist requires)))
(+org--relative-path (+org--read-link-path key (funcall dir-fn))
(funcall dir-fn))
)
:follow
(closure
((dir-fn closure
((plist) (dir-var . org-directory) (key . "org") t) nil
(symbol-value dir-var))
(requires) (plist) (dir-var . org-directory) (key . "org") t)
(link)
(org-link-open-as-file (expand-file-name link (funcall dir-fn)) nil)
)
:face
(closure
((dir-fn closure
((plist) (dir-var . org-directory) (key . "org") t) nil
(symbol-value dir-var))
(requires) (plist) (dir-var . org-directory) (key . "org") t)
(link)
(let*
((path (expand-file-name link (funcall dir-fn)))
(option-index (string-match-p "::\\(.*\\)\\'" path))
(file-name (substring path 0 option-index)))
(if (file-exists-p file-name) 'org-link 'error))
)
)
("attachment" :image-data-fun +org-inline-image-data-fn :follow
org-attach-follow :complete org-attach-complete-link)
("file+sys") ("file+emacs") ("shell" :follow org-link--open-shell)
("news" :follow
#[514 "\301\300\302Q\"\207" ["news" browse-url ":"] 6
"\n\n(fn URL ARG)"]
)
("mailto" :follow
#[514 "\301\300\302Q\"\207" ["mailto" browse-url ":"] 6
"\n\n(fn URL ARG)"]
)
("https" :follow
#[514 "\301\300\302Q\"\207" ["https" browse-url ":"] 6
"\n\n(fn URL ARG)"]
:image-data-fun +org-http-image-data-fn)
("http" :follow
#[514 "\301\300\302Q\"\207" ["http" browse-url ":"] 6
"\n\n(fn URL ARG)"]
:image-data-fun +org-http-image-data-fn)
("ftp" :follow
#[514 "\301\300\302Q\"\207" ["ftp" browse-url ":"] 6
"\n\n(fn URL ARG)"]
)
("help" :follow org-link--open-help)
("file" :complete org-link-complete-file :face
org-roam--file-link-face :export org-roam-server-export-file-id)
("elisp" :follow org-link--open-elisp)
("doi" :follow doi-link-menu :export
#[(doi desc format)
"\b\304\267\202 \300\305 \n\v\206\306\nP$\207\300\307 
\n\v\206\306\nP$\207\310\207"
[format

doi-utils-dx-doi-org-url

doi

desc

#s

(hash-table






size






2






test






eq






rehash-size






1.5






rehash-threshold






0.8125






purecopy






t






data






(html 6 latex 19)






)

"<a href=\"%s%s\">%s</a>" "doi:" "\\href{%s%s}{%s}" nil]
6]
)
)
org-latex-format-headline-function 
'org-latex-format-headline-default-function
org-capture-templates '(("t" "Personal todo" entry
(file+headline +org-capture-todo-file "Inbox") "* [ ] %?\n%i\n%a"
:prepend t)
("n" "Personal notes" entry
(file+headline +org-capture-notes-file "Inbox") "* %u %?\n%i\n%a"
:prepend t)
("j" "Journal" entry (file+olp+datetree +org-capture-journal-file)
"* %U %?\n%i\n%a" :prepend t)
("p" "Templates for projects")
("pt" "Project-local todo" entry
(file+headline +org-capture-project-todo-file "Inbox")
"* TODO %?\n%i\n%a" :prepend t)
("pn" "Project-local notes" entry
(file+headline +org-capture-project-notes-file "Inbox")
"* %U %?\n%i\n%a" :prepend t)
("pc" "Project-local changelog" entry
(file+headline +org-capture-project-changelog-file "Unreleased")
"* %U %?\n%i\n%a" :prepend t)
("o" "Centralized templates for projects")
("ot" "Project todo" entry #'+org-capture-central-project-todo-file
"* TODO %?\n %i\n %a" :heading "Tasks" :prepend nil)
("on" "Project notes" entry
#'+org-capture-central-project-notes-file "* %U %?\n %i\n %a"
:heading "Notes" :prepend t)
("oc" "Project changelog" entry
#'+org-capture-central-project-changelog-file "* %U %?\n %i\n %a"
:heading "Changelog" :prepend t)
)
org-link-translation-function 'toc-org-unhrefify
org-link-elisp-confirm-function nil
org-todo-keyword-faces '(("[-]" . +org-todo-active) ("STRT" . 
+org-todo-active)
("[?]" . +org-todo-onhold) ("WAIT" . +org-todo-onhold)
("HOLD" . +org-todo-onhold) ("PROJ" . +org-todo-project))
org-capture-after-finalize-hook '(org-roam-server-capture-servlet
+org-capture-cleanup-frame-h)
org-latex-format-inlinetask-function 
'org-latex-format-inlinetask-default-function
org-html-format-drawer-function #[514 "\207" [] 3 "\n\n(fn NAME CONTENTS)"]
org-image-actual-width nil
org-ref-get-pdf-filename-function 'org-ref-get-pdf-filename
org-roam-directory "/Users/junwei/github/org/roam/"
org-eldoc-breadcrumb-separator " → "
org-html-format-headline-function 
'org-html-format-headline-default-function
org-entities-user '(("flat" "\\flat" nil "" "" "266D" "♭")
("sharp" "\\sharp" nil "" "" "266F" "♯"))
org-src-window-setup 'other-window
org-confirm-babel-evaluate nil
org-ref-cite-onclick-function 'org-ref-cite-click-helm
org-use-sub-superscripts '{}
org-directory "~/github/org/"
org-preview-latex-image-directory 
"/Users/junwei/.emacs.d/.local/cache/org-latex/"
org-roam-buffer-window-parameters '((no-delete-other-windows . t))
org-ref-notes-function 'orb-notes-fn
org-tags-exclude-from-inheritance '("crypt")
org-html-validation-link nil
org-attach-use-inheritance t
org-agenda-inhibit-startup t
org-agenda-start-on-weekday nil
org-default-notes-file "/Users/junwei/github/org/notes.org"
org-id-locations-file-relative t
org-list-allow-alphabetical t
org-agenda-skip-unavailable-files t
)

-- 
Junwei
CryptoExperts
https://www.cryptoexperts.com


[-- Attachment #2.1: Type: text/html, Size: 195520 bytes --]

[-- Attachment #2.2: Screenshot 2020-12-11 at 18.18.10.png --]
[-- Type: image/png, Size: 148506 bytes --]

^ permalink raw reply	[flat|nested] 4+ messages in thread

* Re: Bug: Orgmode export takes "AC:" as a link keyword [9.5 (nil @ /Users/junwei/.emacs.d/.local/straight/build-27.1/org-mode/)]
  2020-12-11 17:21 Bug: Orgmode export takes "AC:" as a link keyword [9.5 (nil @ /Users/junwei/.emacs.d/.local/straight/build-27.1/org-mode/)] Junwei Wang
@ 2020-12-13  6:05 ` Kyle Meyer
  2020-12-13 17:24   ` Junwei Wang
  0 siblings, 1 reply; 4+ messages in thread
From: Kyle Meyer @ 2020-12-13  6:05 UTC (permalink / raw)
  To: Junwei Wang; +Cc: emacs-orgmode


Junwei Wang writes:

> When exporting orgmode file into HTML file, the exporter "mistakely"
> consider "AC:" as some keyword for linking and the string following
> "AC:" would be something link to.

It looks like org-ref is setting up these links for you. Here's the
relevant bit from the configuration you included:

   org-link-parameters '([...]
   ("Ac" :follow or-follow-acronym :face org-ref-acronym-face :help-echo
   or-acronym-tooltip :export
   #[771 "\211\301>\203\302\303\300A#\207\302\304\226\"\207"
   [("Ac" . "Gls") (latex beamer) format "\\%s{%s}" "%s"] 7
   "\n\n(fn PATH _ FORMAT)"]
   )
   ("ac" :follow or-follow-acronym :face org-ref-acronym-face :help-echo
   or-acronym-tooltip :export
   #[771 "\211\301>\203\302\303\300A#\207\302\304\226\"\207"
   [("ac" . "gls") (latex beamer) format "\\%s{%s}" "%s"] 7
   "\n\n(fn PATH _ FORMAT)"]
   [...]

I suspect the uppercase "AC" being treated as a link despite it not
appearing in org-link-parameters is a case-related bug somewhere in
Org's link handling.  While that's worth looking into, it just shifts
things as far as your original example is concerned: replace "AC" with
"ac", and you have the same issue that you reported.

One way to make Org not treat "ac:foo" as a link is to insert a
zero-width space (with, e.g., <C-x 8 RET 200B>) between "ac" and ":".

  (info "(org)Escape Character")


^ permalink raw reply	[flat|nested] 4+ messages in thread

* Re: Bug: Orgmode export takes "AC:" as a link keyword [9.5 (nil @ /Users/junwei/.emacs.d/.local/straight/build-27.1/org-mode/)]
  2020-12-13  6:05 ` Kyle Meyer
@ 2020-12-13 17:24   ` Junwei Wang
  2020-12-16  6:00     ` Kyle Meyer
  0 siblings, 1 reply; 4+ messages in thread
From: Junwei Wang @ 2020-12-13 17:24 UTC (permalink / raw)
  To: Kyle Meyer; +Cc: emacs-orgmode

Thank you very much for your quick reply.


How can I let orgmode disable some built-in links if it allows? (The 
escape character solution sounds not ideal.)

On 13/12/2020 07:05, Kyle Meyer wrote:
> Junwei Wang writes:
>
>> When exporting orgmode file into HTML file, the exporter "mistakely"
>> consider "AC:" as some keyword for linking and the string following
>> "AC:" would be something link to.
> It looks like org-ref is setting up these links for you. Here's the
> relevant bit from the configuration you included:
>
>     org-link-parameters '([...]
>     ("Ac" :follow or-follow-acronym :face org-ref-acronym-face :help-echo
>     or-acronym-tooltip :export
>     #[771 "\211\301>\203\302\303\300A#\207\302\304\226\"\207"
>     [("Ac" . "Gls") (latex beamer) format "\\%s{%s}" "%s"] 7
>     "\n\n(fn PATH _ FORMAT)"]
>     )
>     ("ac" :follow or-follow-acronym :face org-ref-acronym-face :help-echo
>     or-acronym-tooltip :export
>     #[771 "\211\301>\203\302\303\300A#\207\302\304\226\"\207"
>     [("ac" . "gls") (latex beamer) format "\\%s{%s}" "%s"] 7
>     "\n\n(fn PATH _ FORMAT)"]
>     [...]
>
> I suspect the uppercase "AC" being treated as a link despite it not
> appearing in org-link-parameters is a case-related bug somewhere in
> Org's link handling.  While that's worth looking into, it just shifts
> things as far as your original example is concerned: replace "AC" with
> "ac", and you have the same issue that you reported.
>
> One way to make Org not treat "ac:foo" as a link is to insert a
> zero-width space (with, e.g., <C-x 8 RET 200B>) between "ac" and ":".
>
>    (info "(org)Escape Character")



^ permalink raw reply	[flat|nested] 4+ messages in thread

* Re: Bug: Orgmode export takes "AC:" as a link keyword [9.5 (nil @ /Users/junwei/.emacs.d/.local/straight/build-27.1/org-mode/)]
  2020-12-13 17:24   ` Junwei Wang
@ 2020-12-16  6:00     ` Kyle Meyer
  0 siblings, 0 replies; 4+ messages in thread
From: Kyle Meyer @ 2020-12-16  6:00 UTC (permalink / raw)
  To: Junwei Wang; +Cc: emacs-orgmode

Junwei Wang writes:

> How can I let orgmode disable some built-in links if it allows? (The 
> escape character solution sounds not ideal.)

Sorry, I don't have any other solutions aside from pruning the entries
you don't want from org-link-parameters (see org-link-set-parameters for
function calls that should follow a change in its value) or preventing
them from being added in the first place.

Perhaps someone else knows of another solution and will chime in.


^ permalink raw reply	[flat|nested] 4+ messages in thread

end of thread, other threads:[~2020-12-16  6:01 UTC | newest]

Thread overview: 4+ messages (download: mbox.gz / follow: Atom feed)
-- links below jump to the message on this page --
2020-12-11 17:21 Bug: Orgmode export takes "AC:" as a link keyword [9.5 (nil @ /Users/junwei/.emacs.d/.local/straight/build-27.1/org-mode/)] Junwei Wang
2020-12-13  6:05 ` Kyle Meyer
2020-12-13 17:24   ` Junwei Wang
2020-12-16  6:00     ` Kyle Meyer

Code repositories for project(s) associated with this public inbox

	https://git.savannah.gnu.org/cgit/emacs/org-mode.git

This is a public inbox, see mirroring instructions
for how to clone and mirror all data and code used for this inbox;
as well as URLs for read-only IMAP folder(s) and NNTP newsgroup(s).